Vous êtes sur la page 1sur 66

STUDY NOTE-4

CO-ORDINATE GEOMETRY
4.1 CO-ORDINATES :

Distance between two points P(x1, y1) and Q(x2, y2) is (x 2 - x1 )2 + (y 2 - y1 )2 ………..(i)
Where P,Q lie on the 1 st quadrant of x and y axes.
Y

y 2)
( x 2,
Q

y 1)
x 1,
P(

X
O

P
R
T
Q S
N

X
O K L M
MATHS 4.01
In the above fig. P(x1, y1) and Q(x2,y2) are two points. R is a point on the line PQ. The ratio PQ/RQ
is known as section ratio or R with respect to P and Q.
CO- ORIDINATE GEOMETRY

Let PR = m and co-ordinates of R be (x,y).


RQ n
Now RT = x1–x; QN = x – x2 : PT = y1 – y; RN = y – y2

From the similar triangles PTR and RQN, we have PR = RT = PT


RQ QN RN
x - x y1 - y
Or, m = 1 =
n x - x 2 y - y2

mx 2 + nx1
From the first two fractions, we find x = and from first and third fractions, we get
m+n
my 2 + ny1
y=
m+n
æ mx 2 + nx1 my 2 + ny1 ö
\ the co-ordinates of R are ç , ÷ ………………….(A)
è m+n m+n ø

Note. If m = n, R is the middle point of PQ and hence the co-ordinates of R will be :


æ x 2 + x1 y 2 + y1 ö
ç , ÷
è 2 2 ø

Example (a) : The point C divides the line joining the points A (4, 5) and B (7, –1) internally in the
ratio 1 : 2. Find the co-ordinates of C.
Let (x, y) the co-ordinates of C.

Now x = 1.7 + 2.4 = 7 + 8 = 15 = 5


1+ 2 3 3
1(-1) + 2.5 - 1 + 10 9
And y= = = =3
1+ 2 3 3
\ reqd. co-ordinates of C are (5,3).
Example (b) : Find the co-ordinates of A, which divides equally the line joining B (4, 3) and C (–2, 1)
internally.
Here A divides the BC in the ratio of 1 : 1.
4 + (-2) 4 - 2
x= = = 1, y = 3 + 1 = 2 .
2 2 2
EXTERNAL DIVISION :
PQ m
Let R divide the line joining P (x1, y1) and Q (x2,y2) externally such that = ; let also the co-
RQ n
ordinates of R be (x, y)
æ mx 2 - nx1 my 2 - ny1 ö
Then the co-ordinates of R are ç , ÷ …………………(B)
è m-n m-n ø

4.02 MATHS
Note. This result (B) may be also be obtained by putting n for n and (A) (in case of internal division).
Example (a) : Find the co-ordinates or R, which divides the line joining the points P (8,9) and
Q (_7, 4) externally in the ratio of 4 : 3.
Let (x, y) be the co-ordinates of R,

Now,

Example (b) : If point (3, y) is equidistant from the points (3,3) and (6, 4) then find the value of y.
Let the points A, B and C have respective co-ordinates (3,y), (1,3) and (6, 4).
Now, AB = AC i.e. AB2 = AC2
AB2 = (3 + 1)2 + (y 3)2 = 16 + y2 6y + 9 = y2 _ 6y + 25
AC2 = (3 _ 6)2 + (y _ 4)2 = 9 + y2 8y + 16 = y2 _ 8y + 25
Now y2 _ 6y + 25 = y2 8y + 25 or, y = 0
CENTROID OF A TRIANGLE :
The centroid of a triangle is the intersection of three medians of a triangle.
Let A (x1, y1), B (x2, y2) and C (x3, y3) be the vertices of a triangle ABC, the co-ordinates of centroid

CENTROID OF A TRIANGLE :
The centroid of a triangle is the intersection of three medians of a triangle.
Let A (x1, y1), B (x2, y2) and C (x3, y3) be the vertices of a triangle ABC, the co-ordinates of centroid

(the proof is not shown at present)

MATHS 4.03
4.04
CO- ORIDINATE GEOMETRY

MATHS
Now radius OA = {(7 - 1)2 }+ (- 3 - 5)2 = 100 = 10 units.

3 : Find the co-ordinates of the point which divides the join of (1, 8), (6, 3) in the ratio 2 : 3 internally.
Let (x, y) are the co-ordinates of the point. Hence m = 2 and n = 3.

Internal Division : x = 2.6 + 3.1 = 15 = 3.


2+3 5

y = 2.3 + 3.8 = 6 + 24 = 30 = 6 . \ Co-ordinates = (3, 6)


2+3 5 5

External Division : x = 2.6 - 3.1 = 12 - 3 = -9


2-3 -1

y = 2.3 - 3.8 = 6 - 24 = 18 \ Co-ordinates = (–9, 18)


2-3 -1
1. A (1,2) and B (5,–2) are two given points on the xy plane
are on which C is a moving point such that
the value of the area of the triangle ABC is 12 sq. units. Find the equation of locus of C.
[ICWA (F) June, 2007]
Let C (x, y) be the moving point. Now area of D ABC is

1 [1(- 2 - y )+ 5(y - 2)+ x (2 + 2 )]=12, or 1 (–2 –y + 5y – 10 + 4x)= 12


2 2

or, 1 (4x + 4y –12) = 12 or, 2x + 2y – 6 = 12, or, x + y = 9


2
5. A (– 5, 3) and B (2, 4) are two fixed points. If a point P moves in the (x –y) plane such that
PA : PB = 3 : 2, find the equation to the locus of P. [ICWA (F) June, 2006]

Co-ordinates of P be (x,y) (say). Now PA = (x + 5)2 + ( y - 3) 2 ,

2
PB = (x - 2)2 + ( y - 4) 2 . Since PA/PB = 3 so, PA =9,
2 PB 2 4

( x + 5) 2 + ( y - 3) 2
or, = 9 or, 4 {(x 2 + 10x + 25 + y 2 - 6y + 9}= 9 {(x 2 - 4x + 4 + y 2 - 8y + 16)}
2 2 4
( x - 2) + (y - 4 )
or, 5x2 + 5y2 – 76x – 48y + 44 = 0 (on reduction).
6 : Find the ratio in which the point (11, –3) divides the join of points (3,4) and (7, 11).
Let the ratio of division be 1 : k.

Now, 11 = 7 + 3k or, 7+3k = 11 + 11k or, –8k = 4; or k = - 1


1+ k 2

\ the required ratio = 1 : - 1 or, 2 : 1


2
Hence the point divides externally.
Note.

MATHS 4.05
7. If the point (6, 3) divides the segment of the line from P(4,5) to Q (x, y) in the ratio 2 : 5 find the
co-ordinates (x, y) of Q. What are the co-ordinates of the midpoint of PQ?
CO- ORIDINATE GEOMETRY

Here, 6 = 2x + 5.4 (as m = 2, n = 5)


2+5

Again, 6 = 2x + 20 or, 2x + 20 = 42 or, 2x = 22 or, x =11


7
2 y + 5 .5
Again, 3 = or, 2y + 25 = 21 or, y = – 2
7
\ Required co-ordinates are (11, –2)

Co-ordinates of midpoint of PQ = æç 4 + 11 , 5 - 2 ö÷ or, æç 15 , 3 ö÷


è 2 2 ø è 2 2ø
8. In what ratio is the join of the points (4, –1) and (5, 3) divided by the line x + 3y –8 = 0
[ICWA (F) June, 2004]
Let (x, y) be the intersecting point. Which divides the join of the points (4, –1) and (5, 3) in the ratio
1:k

Now x = 1.5 + 4k , y = 1.3 - k . This point also has in the line x + 3y – 8 = 0 so that
1+ k 1+ k
5 + 4k + 3 × 3 - k or, 5+4x + 9 – 3k = 8 + 8k.
1+ k 1+ k

or, k = 6 , So reqd. ratio = 1 : 6 = 7 : 6.


7 7
SELF EXAMINATION QUESTIONS :
1. Prove that the triangle whose vertices are (–2, 5), (5, –2) and (10, 10) is isosceles.
2. Show that points (6, 6), (2, 3) and (4, 7) are the vertices of a right angled triangle.
3. Prove that the points (4, 3), (7, –1) and (0, 3) are the vertices of an Isosceles triangle.

( )
4. Prove that the points (2a, 4a), (2a, 6a) and 2a + 3a , 5a are the vertices of an equilateral triangle
whose side is 2a.
5. Prove that the points (–3, 5), (6, –1) and (10, 5) are the vertices of a right angled triangle and find
the length of the hypotenuse.
6. Show that the three points (0, 1), (–1, 3) and (2, –3) lie on a right line.
7. Prove that the points (–2, 6), (1, 2), (10, 4) and (7, 8) when joined in order form a parallelogram.
8. Prove that the points (–2, 1), (–1, –3), (3, –2) and (2, 2) are the vertices of a square.
9. Find the co-ordinates of the points which divide the join of
(i) (4, 5) and (7, –5) internally in the ratio of 1 : 2
(ii) (6, 4) and (7, –5) externally in the ratio of 3 : 2. [Ans. (i) (5, 3); (ii) (9, –23)]
10. Find the co-ordinates of the centroid of the triangle whose vertices are (1, 4),, (5, 3) and (6, 2)
[Ans. (4, 3)]

4.06 MATHS
11. The centroid of a triangle ABC is at the point (1, 2). A is that point (1, 2). and B is the point (2, 3).
Find the co-ordinates of C. [Ans. (–2, 2)]
12. Find the ratio in which the point (5, 3) divides the join of the points (4, 5) and (7, –1).
[Ans.1: 2]
13. Find the area of a triangle whose vertices are (4, 1), (–4, 4) and (8, –3). [Ans. 10 sq. units]
14. Prove that the following three points are collinear :
(i) (4,2), (7, 5) and (9, 7) (ii) (1, 4), (3, –2) and (–3, 16)

15. prove that (a, 0), (0, b) and (1, 1) will be collinear, if : 1 + 1 = 1
a b

LOCUS :

If a point in a plane obeying some geometrical condition (s), then the path so traced is known as locus
of that point. In other words, the curve obtained by joining number of points obeying the geometrical
conditions(s) is the graph of the locus.
For instance, the locus of a point which moves in a plane keeping a constant distance from a fixed (or
given) point is a circle. Again the locus of a point that moves keeping a constant distance from a fixed
line is another straight line parallel to the given straight line.
P and Q are two fixed points in a plane. If in the same plane, another point A moves keeping a fixed
equal distance from the given points P and Q, then the locus of A is straight line.
Note. The fixed rule is to keep a fixed equal distance from the given points.
Here (see Fig.)
P

A1 A2 A3
A

Q
PA1 = QA1 PA2 = QA2 PA3 = QA3 (i.e. the distance is always same)
Locus of a point may be a straight line or a curve line.
Equation of locus :
By the given condition if a point moves, then there exists some algebraic relation (or equation)
involving the co-ordinates of that moving point. Such a relation is called the equation of the locus.
The co-ordinates of the moving point are represented by x and y.
The algebraic relation is true for all points lying on the locus but not for any outside points.
General rule for finding the equation of locus :
MATHS 4.07
General rule for finding the equation of locus :
(i) Find the condition (or conditions) that the moving point obeys.
(iii) Express the given condition by the help of x and y and then simplify.
(iv) After simplification, find the algebraic relation with x and y.
(v) The obtained relation will be equation of locus. The co-ordinates of the moving point (i.e. x,
y) are known as current co-ordinates. The equation of the locus is generally expressed in
terms of current co-ordinates.
SOLVED EXAMPLES :
1. A point P moves in a plane keeping equal distance from two fixed points A(–4, 1) and
B (2, 5). Find equation of that locus of the moving point.
Let us consider any position P of the moving point having co-ordinates (x, y). Now we will have
distance PA = distance PB or, PA 2 = PB 2.
Now, PA 2 = (x + 4) + (y – 1)2 = x2 + 4x + 16 + y2 –2y +1 = x2 + y2 +4x – 2y + 17
Again, PB 2 = (x – 2)2 + (y –5)2 = x2 – 4x + 4 + y2 – 10y + 25
Now, x2 + y2 + 4x –2y + 17 = x2 + y2 –4x –10y + 29
or, 8x + 8y = 12 or, 2x + 2y = 3
2 : Find the locus of a point which moves so that the sum of the squares of its distances from the point
(2, 0) and R (–2, 0) is always equal to 40.
Let Q (2, 0) and R(– 2, 0) and P (x, y) be any point on the locus.
By condition, PQ 2 + PR 2 = 40
Or, (x – 2)2 + (y – 0)2 + (x + 2)2 + (y – 0)2 = 40 Or, x2 + y2 = 16 (on reduction)
2 2
\ Required education of locus is : x + y = 16

3. Find the locus of a point which moves such that its distance from the point (4, 0) is 2 times of its
distance from the y-axis.

Let P(x, y) be the point. Now distance of P from (4, 0) is ( x - 4) 2 + (y - 0 )2 ; again distance of P

from y-axis is x, By question, we find :

( x - 4) 2 + (y - 0 )2 = x 2 or, (x – 4)2 + y2 = 2x2

x2 + 8x – y2 = 16, which is the required equation of locus.


4 : A point (x, y) moves in such a way that the differences of its distances from A (1, 4) and B(1, –4)
is always equal to 6. Find the equation to the locus of P. [ICWA (F) June, 2004]
Let P (x, y) be the moving point. Now, PA – PB = ± 6
2 2 2 2
Or, ( x - 1) / ( y - 4 ) - (x - 1) / (y + 4 ) = ±6

2 2 2 2
Or, ( x - 1) + (y - 4 ) = ± 6 + ( x - 1) + (y + 4 )

Or, (x – 1)2 + (y – 4)2 = 36 + (x – 1)2 + (y + 4)2 ± 12 (x - 1)2 + (y + 4 )2

Or, –16y – 36 = ± 12 (x - 1)2 + (y + 4 )2


4.08 MATHS
Or, – (4y + 9) = ± 3 (x - 1)2 + (y + 4 )2

Or, 9x2 – 7y2 – 18 x + 72 = 0, which is the eqn. of locus.


SELF EXAMINATION QUESTIONS :
1. A point moves in such a way that
(i) The sum of distances from axes is 10.
(ii) The distance from x-axis is equal to its distance from the point (1,1).
(iii) Sum of its distances from the points (3, 0) and (– 3, 0) is always 10 unit.
[Ans. (i) x + y = 10, (ii) x2 – 2x – 2y + 2 = 0 (iii) 12x2 – 4y2 = 75.]
2. Find the equation locus of a point such that its distance from y axis is double of its distance from
the point (2,3) [Ans. 3x2 + 4y2 –16x – 6y +32 = 0]

3. Find the locus of a point the moves so that its distance from the point (4, 0) is 2 times of its
2 2
distance from the y-axis. [Ans. x + 8x – y = 16]
4. A moving point is collinear with the point (2, –3) and (5, 7). Find the equation of locus of that
point. [Ans. 10x – 3y = 29]
1. Find the equation of the perpendicular bisector the join of the point ( –1, 2) and (5, 4)
[Ans. 3x + y = 9]
GRADIENT (or Slope) :
The gradient of a straight line (not perpendicular to x-axis) is the increase of ordinate per unit
increase in abscissa at any point in the same line.
increase in ordinate
\gradient =
increase in abscissa
Note. In measuring gradient (or slope) of a line, the abscissa and ordinates are to be taken in the same

units. A road of 15% slope æç i.e., 15 slope ö÷ means for every 100 m distance horizontally, the rise of
è 100 ø
the road is 15 m.
Example : Find the slope of the median AD of the triangle ABC, where D is middle point of the base
BC. The co-ordinates of A, B and C are respectively (4, 1), (– 4, 4) and (8, –3).

Co-ordinates of D = æç 8 - 4 , - 3 + 4 ö÷ = æç 2 , 1 ö÷
è 2 2 ø è 2ø
1 -1
Slope of AD = 2 ; here y 2 = 1 , y1 = 1, x2 = 2, x1 = 4
2-4 2
-1
= 2 =1
-2 4
(Students may now find the slope of the other two medians.)
Use of Slope :
In determining parallel and perpendicular lines, the following two results may be remembered :

MATHS 4.09
1. If two lines are parallel, then their slopes are equal and conversely, if the slopes of two lines are
found to be equal, then two lines are parallel.
CO- ORIDINATE GEOMETRY

2. If two lines are perpendicular, then the product of their slopes is –1 and conversely, if the product
of the slopes is –1 then the two lines are perpendicular.
(Student will be acquainted with the above result again in the next chapter, in the part of conditions of
parallelism and perpendicularity of two lines.)
SOLVED EXAMPLES :
1 : Verify by using concept of slope that the points A (3, – 2), B (– 1, 1), C (– 5, 4) are collinear.
1 - ( -2) 1 + 2
Slope of AB = = =-3
-1- 3 -4 4
4 - (-2) 4 + 2
Slope of AC = = =-6 =-3
-5-3 -8 8 4
We find that the two lines AB and AC pass through the same point A and have the same slope. So the
two lines coincide, i.e., A, B and C are collinear.
Note. We may also find the result by using distance formula, discussed earlier.
2 : Show that the points A (0, 3) B ( –2, 1) and C (–1, 4) are the vertices of right-angled triangle.

Slope AB = (= m1 say) = 1 - 3 = -2 = 1,
-2-0 -2

Slope AC (= m2 say) = 4 - 3 = 1 = -1
-1- 0 -1
Now we find m1 × m2 = –1 so, AB is perpendicular to AC at the point A.
\ ABC is a right-angle triangle.
(by using distance formula also, we may get the result.)
3 : Find the slope of the line : (i) parallel to and (ii) perpendicular to the line joining the points P (2, 5)
and Q (–4, 3).
(i) The slope of a line parallel to PQ will be the same as the slope of PQ.

Now slope of PQ = 3 - 5 = -2 = 1
-4-2 -6 3
(ii) In this case (Slope of line) × (slope of PQ) = – 1
or slope of line × 1/3 = –1.
Hence, slope of the line = –3.
SELF EXAMINATION QUESTIONS :
1. Verify (using slope] that the points A (0, 1), B(1, – 4) and C(–2, 11) are collinear.
2. Verify that the points P (a, b + c), Q (b, c + a) and R (c, a + b) are collinear.
3. Find the slope of the median AD of the triangle having vertices A (7, –5), B (–2, 5), C (4, 6) and
D is the middle point BC. [Ans.–21/12]
4. Show (by using slope) that the points A (6, 6), B (2,3), and C (4, 7) are the vertices of a right
angled triangle.
5. If the three points (x, y), (5, –2) and (3, –4) are collinear, show that x – y –7 = 0
4.10 MATHS
OBJECTIVE QUESTIONS :

1. Find the distance between the points (2, –2) and (–1, 2) [Ans. 5 units]
2. The distance between the points (2, –1) and (t, – 4) is 5, find t [ Ans. 6, – 2]
3. What point on x axis equidistant from the point P (– 3, 4) and Q (7, 6). [Ans. (3, 0)]
4. For the triangle having vertices A(4, 3), B (7, –1) and C(9, 3). Find the length of the median

AD. [Ans. 2 5 units]


5. Find the co-ordinates of the centroid of the triangle having vertices (1,4), (5, 3) and (6,2)
[Ans. (4, 3)]
6. Find the ratio in which the line segment joining A (2,4) and B(–4, 6) is divided by y-axis.
[Ans. 1 : 2]
7. Find the ratio in which C(2,9) divides the line segment of A (8, 12), B(–2, 7) internally.
[Ans. 3 : 2]
8. If the points A (4,2) B (7,5), C(x, 7) are collinear. Find x [Ans. 9]
9. A(–4, 6) and B(6, 2 b) are two points and the co-ordinates of the middle point of AB are (1, 3),
find b . [Ans. 0]
10. The centroid of a triangle ABC is at the point (1, 2). A is the point (3,1) and B is the point (2, 3),
find C [Ans.( –2, 2)]

11. Find the distance of the point (4, –2) from the origin. [Ans. 2 5 units]

12. Find the area of the triangle having vertices (0,0) (1,0) and (0,1). [Ans. 1 sq. units]
2
13. If the point (5, y) divides the line joining the points (4, 2) and (7,5) internally, find y
[ICWA (F) Dec2000] [Ans. 2]
14. Find the point which divides the joining of the points (3, 5) and (6, 8) by a straight line in the ratio
2: 1 internally. [ICWA(F) June, 2006] [Ans. (5.7)]
15. Find the co-ordinates of the point that divide the line segment joining the point (5, 8) and (6,3) in
the ratio 2: 3 externally. [ICWA (F) June, 2005] [ Ans. (3, 18)]
16. The point P divides the line joining the points M(4, 5) and N(7, –1) internally in the ratio 1 : 2.
Find the co-ordinates. [ICWA(F) Dec. 2004] [Ans. (5,3)]
4.2 THE STRAIGHT LINE :
Introduction : 4.2 THE STRAIGHT LINE :
10.1.1.1.1.1.1.1.1
If a point moves in a plane without changing its direction, then its locus is a straight line.
A straight line can be expressed as an equation of the first degree of linear equation in x and y.

MATHS 4.11
Equations of lines parallel to Axes :
Let AB be a straight line parallel to y-axis and at a distance ‘a’ from y-axis. For any point P (x, y) on
AB its abscissa is always a, i.e., x = a. Now any point not lying on AB does not satisfy this condition
x = a.
\ the equation of a straight line parallel to y-axis at a distance of a x = a.
Note. : --
1. If, a = 0, then AB coincides with y-axis and its equation is x = 0.
Similarly, the equation of a straight line parallel to x-axis and at a distance b will be y = b. if again
b = 0, the same line coincides with x-axis and its equation is y = 0.
2. Here the distance a and b may be positive or negative.
Y

x=3
3

X´ X
(0,0)
–4

x = –4


Example. If a = 3 (in the first case) then we get x =3, the line parallel to y-axis is shown in the Fig.
For b = –4, we get a line parallel to x-axis (shown in the same figure).
Equation of a line through two given points :
Let A (x1, y1), b (x2 , y2) be the given points and let C (x, y) be any point on AB.
In the following Fig. From similar triangles ACT and ABS, we have

Y
B (x2,y2)
)
x,y
) C(
x 1,y
1

A(
S T

O X
P Q R

AT = CT x - x1 y - y1 x - x1 y - y1
or, = writing in order, =
AS BS x 2 - x1 y 2 - y1 x1 - x 2 y1 - y 2
Which is the required equation.
4.12 MATHS
y
Cor. If any one point coincides with origin, say, x1 = y1 = 0, then the equation reduces to x = .
x 2 y2

y 2 - y1
Alternatively, the gradient of the line joining A to B is and the gradient of the line joining A
x 2 - x1

y - y1
to C is .
x - x1

y 2 - y1 y - y1 x - x1 y - y1
We know = (as two gradients or slopes are equal) or, = or
x 2 - x1 x - x1 x 2 - x1 y 2 - y1

x - x1 y - y1
= which is the equation.
x1 - x 2 y1 - y 2
INTERCEPT FORM :

B (0,b)
P (x,y)

A (a,0)
X
O Q

Let the line AB make intercepts OA = a, on x axis and OB = b on y axis, let P (x, y) be any point on
AB.
AQ AO
In the fig.. from the similar triangles APQ and ABO, we find = or , a - x = a
PQ BO y b

y
or, x + = 1 which is the required equation.
a b
Note. Due attention must be given to signs of a and b. Here a and b have same sign in the direction of
the axes.
Alternatively, the co-ordinates of A and B are (a, 0) and (0, b).
y-0 y
By the previous method; we get, x - a = or , x + = 1.
a-0 b-0 a b
EQUATION OF A LINE HAVING A GIVEN GRADIENT AND PASSING THROUGH A GIVEN POINT

(OR POINT SLOPE FORM)

Y B

P (x,y)

P1 (x1,y1)

X
O A
MATHS 4.13
Let AB be a straight line passing through P1 (x1 , y1), having a gradient m. If P (x, y) be an arbitrary
point on the line, then
CO- ORIDINATE GEOMETRY

y - y1
m= or, y – y1 = m (x – x1) [which is the required equation]
x - x1
Note. Here m is variable, and varies with different line through P.
GENERAL EQUATION OF A STRAIGHT LINE :
An equation of the form ax + by + c = 0 where a and b are not simultaneously zero always represents
a straight line.
Change of Form :

The equation ax + by + c = 0 may be written as x + y = 1, the intercepts on the axes being


-c -c
a b

- c and - c
a b

Tangent Form. Ax + by + c = 0 or y = – a x - c , the slope is - a and intercepts on y-axis is


b b b
– c/b.
SOLVED EXAMPLE :
Example 1: Find the equation of the straight line passing through the points :
(i) (3, –4) and (1, 2); (ii) (5, 1) and (– 3, 7).
(i) x1 = 3 , y1 = – 4 and x2 = 1, and y2 = 2.
x - x1 y - y1
From the equation =
x1 - x 2 y1 - y 2

y - ( -4) y+4
We get x - 3 = or, x - 3 = or, 3x + y – 5 = 0.
3 -1 - 4 - 2 2 -6
y-4
(ii) Similarly, x - 5 = , or, 9x + 8y = 29.
5+3 -2-7

Alternatively, (with the help of Gradient) :


-( -4) 6
(i) Gradient of the line m = = = -3 \ the required equation y – y1 = m(x – x1)
1 - 3 -2
i.e, y – (– 4) = –3 (x – 3) or, y + 3x – 5 = 0.
7 - ( -2)
(ii) here m = =-9 9
\ The eqn. is y – (–2) = - (x - 5) or, 9x + 8y = 29.
-3-5 8 8

4.14 MATHS
2. Find the equations of diagonals of the rectangle formed by the lines x = 2, x = 5, y = 2 and
y=5 [ICWA (F) June, 2005]
From the adjoining figure we are to find the equation of the lines AC and BD
Y
D (2,5) C (5,5)
y=5
x=2 x=5
A B y=2
(2,2) (5,2)
X
O

y-5
Equation of AC is x - 5 = or, x = y
5-2 5-2
y-5
Equation of BD is x - 2 = or, x + y = 7.
2-5 5-2
3 : Express the equation 3x + 4y + 5 = 0 in the
(i) intercept; and (ii) tangent form.

x y
(i) 3x + 4y + 5 = 0 may be written as + = 1 the intercept on the axes being
-5 / 3 -5 / 4

- 5 and - 5 .
3 4
3 5
(ii) From 3x + 4y + 5 = 0 we get 4y = –3x – 5 or, y = - x - here m (= slope) is - 3 and
4 4 4

intercept (= c) is - 5 on y-axis.
4
4. Find the equation of a straight line (i) passing through the point (– 2, 5) and parallel to x-axis and
(ii) passing through the point (– 3, 4) and parallel to y-axis.
(i) As the straight line is parallel to x-axis, let the equation be y = b. Since this line passes through the
point (– 2, 5), so we have 5 = b.
\ the reqd. equation is y = 5.
(ii) Let the equation be x = a. Since it passes through (– 3, 4) so –3 = a.
\ the reqd. equation is x = – 3, i.e., x + 3 = 0.
5. (i) Find the equation of the line joining the points (3, –4) and (1, 2) and hence find the co-ordinates
of the points on the axes when the line meets.
(ii) Find the equation of the line joining the points A (3, – 4) and B (1, 2) and hence show that the
three points A (3, – 4), B (1, 2) and C (2, – 1) are collinear.
(i) equation of the line joining the points (3, –4), (1, 2) is 3x + y = 5 (se Ex. 1).

MATHS 4.15
y
Now the intercept form of the above equation is 3x + = 1 (dividing by 5 in both sides) or
5 5
x + y =1
\ the co-ordinates of the intersecting points of the line on the x and y axes are
5/3 5

respectively æç 5 , 0 ö÷ and (0, 5).


è3 ø

(ii) Equation of the line AB is x - 3 = x + 4 or, y + 3x = 5.


3 -1 - 4 - 2

Slope of AB 2 + 4 = 6 = -3 ,, slope of AC = -1 + 4 = 3 = -3
1- 3 - 2 2 - 3 -1
Since slope of AB = Slope of AC, so the points A, B, and C are collinear.
6. Show that the points (1, 4), (3, –2) and (–3, 16) are collinear and hence find the equation of the line
where the three points lie.
é x - x1 y - y1 ù
Equation of a line passing through the points (1, 4) and (3, –2) êfrom =
ë x1 - x 2 y1 - y 2 úû

y-4 y-4
Is x - 1 = or, x - 1 = or 6(x –1) = – 2 (y –4) or, 3x + y = 7.
1- 3 4 + 2 -2 6
Now if the third point lies on this line, then it must satisfy the equation. Putting, x = –3, y = 16, we
find
L.H.S. = 3(3) + 16 = –9 + 16 = 7 = R. H. S.
Hence the three points are collinear.
7. Find the equation of a line passing through the point (1, 3) and parallel to the join of the points
(– 2, 5) and (4, 7).

Slope of the line joining the points (–2, 5) and (4, 7) is m = 7 - 5 = 2 = 1 ,


4+2 6 3
Equation of a line passing through the point (1, 3) is y – 3 = m (x – 1). Since, this line is parallel to the
line joining the given points, so m = 1/3.
\ reqd. equation is y – 2 = 1/3 (x –1) or, 3(y – 3) = (x – 1) or, 3 ( y – 3) = x – 1 i.e., 3y – x = 8.
8. Find the equation of the straight line which passes through the point (3, –4) and whose intercepts
on the axes are equal in magnitude but opposite in sign.
Let the intercept be a.
y
Now x + = 1 . Since the line passes through (3, – 4) we have 3 + 4 = 1 , i.e., a = 7.
a -a a a
9. A straight line passes through the point (–4, 9) and is such that the portion of it intercepted between
the axes is divided at the point in the ratio of 3 : 2. Find the equation.
y
Let the required equation be x + = 1
a b
The co-ordinates of the points where this line meets the axes are (a, 0) and (0, b). now co-ordinates of
the point which divides the line joining these two in the ratio of 3 : 2
4.16 MATHS
3. 0 + 2. a 3 . b + 2. b
i.e.
Are and ; i.e. 2a and 3b
3+ 2 3+ 2 5 5
Now this is the point which has co-ordinates –4 and 9.

So, 2a = – 4 and 3b = 9; i.e., a = –10 and b = 15.


5 5

Hence, the required equation is x + y =1; i.e. 3x – 2y + 30 = 0.


- 10 15
10. A firm produces 200 units of a product for a total cost of Rs. 1730 and 500 units of the product for
a total cost of Rs. 97. Assuming the cost curve to be linear, derive the equation of this line and use it
to estimate the cost of producing 400 units of the product.
The linear relation between the product of the firm and the total cost is equation of a line joining the
points (200, 730) and (500, 970).
x - x1 y - y1
We know that the equation of a line joining the points (x1 , y1) and (x2, y2) is : =
x1 - x 2 y1 - y 2

y1 - y 2
i.e. y – y1 = ( x - x1 ) [Here, x1 = 200, x2 = 500, y1 = 730, y2 = 970 ]
x1 - x 2

or, y – 730 = 730 - 970 (x - 200 )


200 - 500

y – 730 = -240 (x - 200) = 4 (x - 200 ) or, 5 (y –730) =4 (x – 200)


- 300 5
or, 5y – 3650 = 4x – 800 or, 5y = 4x – 2850.
Now for x = 499 units, we get 5y = 4 ×400 + 2850 = 4450 or, y = 890
Therefore, required cost is Rs. 890.
SELF EXMENATION QUESTIONS :
1. Find the equation of the straight line which passes through the line point (2, –4) and makes
intercepts on axes:
(i) equal in magnitude and of same sign.;
(ii) equal in magnitude but opposite in sign. [Ans. (I) x+y+2 = 0; (ii) x – y = 6]
2. A straight line passes through (1, –2) and has a gradient 1/3, find its equation. Find also the
intercepts cut off by the line from the axes. [Ans. x –3y = 7; a = 7, b = –7/3]
3. Find the equation of the straight line which passes through the point (5, 6) and has intercepts on
the axes equal in magnitude but opposite in sign. Find also the co-ordinates of the point at which
the ordinate is double the abscissa. [Ans. x – y +1 = 0; 1, 2]
4. Find the equation of the straight line making an intercept on the x-axis and passing through the
point (2, 3). [Ans. 3x + y = 9]
5. A straight line passes through the point (–5, 4) which divides the segment of the line intercepts
between axes in the ratio 1 : 2, find the equation. [Ans. 5y – 8x = 60]
6. Find the equation of the straight line which passe though (– 4, 3) and is such that the portion of it
between the co-ordinate axes is divided by the point in the ratio of 5 : 3. [Ans. 20y –9x = 96]
MATHS 4.17
7.
3. In what ratio is the line joining the points (1, 3) and (2, 7) divided by the line 2x + y = 9.
[Ans.2 : 1]
CO- ORIDINATE GEOMETRY

8.
4. Find the equation of a straight line passing through ( –2, 5) and parallel to x axis.
[ICWA (F) June, 2004] [Ans. x + 2 = 0]
9.
5. Show that the points (3,1), (5, –5) and (– 1, 13) lie on a line and hence find the equation of that
line. [Ans. y + 3x = 10]
10.
6. Find the equation of the line joining the points (3, 6) and (2, –5). Hence show that the points
(3, 6), (2, – 5) and (0, –27) are collinear. [ICWA (F) June, 2000] [Ans. 11x –y =27]
11.
7. A firm produces 59 units of a good for Rs. 320 and 80 units for or Rs. 380. Supposing that the cost
curve is a straight line, estimate the cost of producing 10 units. [Ans. Rs. 240]
12.
8. A firm invested Rs. 10 million in a new factory that has a net return of Rs. 50,000 per year. An
investment of Rs. 20 million would yield net income of Rs. 2 million per year. What is the linear
relationship between investment and annual income? What would be the annual return on an
investment of Rs. 15 million? [Ans. 3x = 20y + 2,00,00,000 : Rs. 12,50,000]
13.
9. The expense of a boarding house are partly fixed and partly vary with the number of boarders. The
charge is Rs. 70 per head when there are 25 boarders and Rs. 60 per head when there are 50
boarders. Find charge per head when there are 50 boarders. Find the charge per head when there
are 100 boarders. [Ans. Rs. 55]
CONDITIONS OF PARALLELISM AND PERPENDICULARITY OF TWO STRAIGHT
LINES :
Parallel :
(i) Two lines y = m1x + c1, y = m2 x + c2 will be parallel if m1 = m2
a1 b1
(ii) Again two lines of the form a1 x + b1y + c1 = 0 and a2x + b2y + c2 = 0 will be parallel if =
a 2 b2

Perpendicular :
(i) Two lines y = m1x + c1, y = m2x + c2 will be perpendicular to each other if m1m2 = – 1
(ii) Again lines a1x + b1y + c2 = 0, a2x + b2y + c2 = 0 will be perpendicular to each other if
a1a2 + b1b2= 0
10.1.1.1.1.1.1.1.1 Working Rule :
a1 b1
For Parallel. The line a1x +b1y + c2 = , a2x + b2 y +c2 = 0 are parallel, if = , i.e. a1 = a2k ; b=1 b2kb2 k
a 2 b2
(where k is constant).
æ c ö
Putting in1st eqn. a2kx + b2 y + c1 = 0 ; ç c¢ = 1 ÷ .
è kø

Thus equation of two parallel lines differs only in constant term.


For Perpendicular. Now for the lines. a1x + b1y + c1 = 0, and b1x – a1y + c2 = 0

4.18 MATHS
a b a b
We get, m1 = - 1 , m 2 = 1 and, m1m 2 = 1 ´ 1 = -1 which shows that the lines are
b1 a1 b1 a1
perpendicular.
Example : Given a straight line 2x + 5y + 6 = 0. Now any line parallel to it is 2x + 5y + k = 0 and
perpendicular to it is 5x – 2y + k1 = 0, or –5x + 2y + k´ = 0
Note. For external angle between two lines; y = m1x + c1 and y = m2x + c2 :
m1 - m 2
Tan (angle between two lines) = ±
1 + m1m 2
Point of Intersection of two Straight lines :
Let the two lines be a1x + b1y + c1 = 0; a2x + b2y + c2 = 0.
If now (x1 , y1) be their point of intersection, then both the lines will pass through the point,
i.e. (x1 , y1) will satisfy the above equation. By solving equations, (x1, y1) can be obtained.
Condition of Concurrence of three lines :
a1 x + b1y+ c1 = 0
a2 x + b2y+ c2 = 0
and a3 x + b3y+ c3 = 0
The point of intersection of first two lines will satisfy the third line.
Line through the Intersection of two given lines :
Let the two lines be a1 x + b1y+ c1 = 0
a2 x + b2y+ c2 = 0
The equation a1 x + b1y+ c1 + k( a2 x + b2y+ c2 )= 0,
where k is an arbitrary constant, represents any straight line such that any value of x and y that
satisfy this equation.
Example 1 : Find the equation to the straight line which passes through the point (5, –3) and is
parallel to the line 7x + 9y –11= 0.
First Method
Any line parallel to 7x + 9 y – 11 = 0 may be written as 7x + 9y + k = 0……(i)
Again this line passes through (5, –3); so 7.5 – 9.3 + k = 0 i.e. k = – 8.
The required equation is 7x + 9y – 8 = 0.
Note. Here, any line parallel to the given line is taken first. Then we choose one line which passes
through the given point. Eq. (i) represents different lines (for k different values) but all parallel to the
given line, out of which only one line passes through (5, –3) and k = – 8 corresponds to this particular
line.

This given line can be written as – y = - 7 x + 11 ………….(i) i.e. gradient = = - 7


9 9 9
Now any line passing through (5, –3) is y + 3 = m (x – 5)

MATHS 4.19
4.20
CO- ORIDINATE GEOMETRY

MATHS
5 28
y=- x+ or, 3y + 5x = 28.
3 3
Example 3. Find the equation to the straight line passing through the point ( –4, 7) and the point of
intersection of the lines 3x + 4y – 5 = 0 and 4x – 5y + 7 = 0
The equation of a line passing through the intersection of the lines 3x + 4y – 5 = 0, 4x – 5y + 7 = 0 is
3x + 4y – 5 + k (4x – 5y + 7) = 0
Again, since this line passes through the point (– 4, 7), we get
3 (– 4) + 4.7 – 5 + k [4. (– 4) – 5 . 7 + 7] = 0
Or, –12 + 28 – 5 + k (– 16 – 35 + 7) = 0
11 1
or 11 - 44k = 0 or, k = = .
44 4
1
The equation is 3x + 4y – 5 + (4x - 5x + 7 )= 0
4
or, 4(3x + 4y – 5) + 4x – 5y + 7 = 0
or, 12x + 16y – 20 + 4x – 5y + 7 = 0
or, 16x + 11y = 13.
Example. 4 : Find the equation of the straight line passing through the point of intersection of the
lines 2x + y = 4 and x – y + 1= 0 and is perpendicular to the line 3x – 5y + 7 = 0
[ICWA (F) June, 2007]
Solving 2x + y = 4 ………….(i), x – y + 1 = 0…………..(ii) we have from (i)
2x + (x + 1) = 4 or, 3x = 3, or, x = 1, From (ii), y = x +1 = 1+1=2
\ Intersecting point is (1, 2). Now any line perpendicular to 3x – 5 y + 7 = 0 is –5 x – 3y + k = 0.
This line passes the point (1, 2). So – 5× 1 – 3 × 2 + k = 0 or, k = 11.
\ reqd. eqn. is – 5x – 3y + 11 = 0 i.e. 5x + 3y = 11.
Example. 5 : Find the equation of the line joining points of intersection of 2x + y = 4 with
x – y + 1 = 0 and 2x – y –1 = 0 with x + y – 8 = 0 [ICWA (F) Dec. 2003]
weget
Solving 2x + y = 4 and x – y +1 = 0, we set x = 1, y = 2, and solving 2x – y–1 = 0 and x + y – 8 = 0
y-2
we have x = 3, y = 5 the equation of line passing through the points (1, 2) and (3, 5) is x - 1 =
1- 3 2 - 5
y-2
or, x - 1 = or, 3x – 2y + 1 = 0.
-2 -3
FEW RESULTS :
ax1 + by1 + c
(A) Length of Perpendicular from (x1 , y1) to a line ax + by + c = 0 is
a 2 + b2
(B) Sign perpendicular. If only the length of the perpendicular is to be determined, the sign of
perpendicular may be ignored.
ax1 + by1 + c
But in case when sign is to be taken into account, perpendicular from (x1, y1) is + ,
a 2 + b2

MATHS 4.21
When the point (x1, y1) is on the origin side, otherwise sign is negative.
In order to determine the sign make the constant term, (c) of the given line always positive and hence
CO- ORIDINATE GEOMETRY

apply the above formula.

Example 5 : Find the perpendicular distances from (1,1), (3, – 5) to the line x – 2y = 4
[ICWA (F) Dec. 2004]
x – 2y – 4 = 0 may be written as –x + 2y + 4 = 0 (c is made positive). Now perpendicular distance

from (1, 1) is - 1 + 2.1 + 4 = 5


(1 + 4)
Again perpendicular distance from (3, –5) is - 3 + 10 + 4 = - 9
(1 + 3) 5

It seems that the two points are on two sides of the given line.
SELF EXAMINATION QUESTIONS :
1. . The straight line joining the points (2, y), (4,1) is perpendicular to the straight line joining the
points (7, –2), (3, 2). Find the value of y. [Ans. –11]
2. Co-ordinates of the vertices of a triangle ABC are respectively (3, 4), (9, 6) and (5, 12). Find the
slope of the median AD of that triangle. [Ans. 5/4]
3. (i) Show that the lines 12x – 9y = 1 and 8x – 6 y + 6 = 0 are parallel.
(ii) Show that the lines ax + by + c = 0 and kax + kby = 0 are parallel.
4.If the lines 2x + 3y + 1 = 0, 6x + ay + 5 = 0 and 4 – 12y + bx = 0 are parallel to each other, then find
the values of a and b. [ Ans. 9, –8]
Find the equation of a straight line passing through origin, and
(i) parallel, (ii) perpendicular to 2x + 3y + 1 = 0 [Ans. (I) 2x + 3y + = 0 (ii) 3x – 2y = 0 ]
1. Find the equation of a straight line passing through origin, and
(i) Parallel, (ii) perpendicular to px + qy + r = 0. [Ans. (i) px + qy = 0, (ii) qx – py = 0]
7. Find the equation of a straight line that passes through :
(i) the point ( – 2, 1) and parallel to 4x – 2y – 3 = 0 [ICWA (F) Dec. 2000]
(ii) the point (– 1, 4) and parallel to 2x + 5y = 1
(iii) the point (4, –3) and perpendicular or x + 2y +7 = 0
(iv) the point (–1, 2) and perpendicular to 3 x + 4 y = 5
(v) the point (3, 4) and perpendicular to 4x – 3y + 1 = 0
(vi) the point (3, 4) and parallel to the line joining the points (1, 3) and (2, 7)
(vii) the point (3, –6) and perpendicular to the line joining the points A (4, 1) and
B (2, 7) [ICWA (F) June, 98]
[Ans. (I) 4x – 2y + 10 = 0 (ii) 2x + 5y + 18 = 0, (iii) y – 2x +11 = 0, (iv) 4x – 3y + 10 = 0,
(v) 3x + 4y = 25, (vi) 4x – y = 8, (vi) 4x – y = 8, (vii) x –2y = 15]
8. Find the point of intersection of the following pair of lines :
(i) x + 5y + 3 = 0; 5x – 2y = 12
4.22 MATHS
(ii) 2 x – 3y + 11 = 0 ; 3x – 4y + 13 = 0 [ Ans. (i) 2, –1; (ii) 5, 7]
9. Show that the following lines are concurrent and find also the co-ordinates of the meeting :

(i) 2x – 7y + 10 = 0; 3x – 2y = 1; x – 12 y + 21 = 0. éAns. 27 , 32 ù
êë 17 17 úû
(ii) y – x + 1 = 0; 2y – x – 1 =m 0 , 7x – y – 19 = 0, [Ans. 3, 2]
10. For what value of m the three lines
2x – 3y + m = 0, 3x – 4y –1 = 0 and 4x – 5y – 2 = 0 are concurrent. [Ans. 0]
11. Show that the medians of the triangle formed by the vertices (3, 2), (1, –1) and (– 19, –9) are
concurrent.
12. Find the equation of a straight line which passes through the intersection of the lines :
(i) x + 5 y + 3 = 0 and 5x – 2y = 12 and parallel to y = 2x + 1. [Ans. 2x – y – 5 =0]
(ii) x + 2y + 3 = 0 and 3x + 4y + 7 = 0, and parallel to y = –5/8x. [Ans. 5x + 8y + 13 = 0]
(iii) x + 2y = 0 and y + 4x +7 = 0, and perpendicular to 3 x = y. [Ans. x + 3y = 1]
13. Prove that the straight line given by the equation (3x –4y +7) + k(5x – 2y – 7) = 0 passes through a
fixed point for all values of k and find that point.
[hints : solve 3x – 4y + 7 = 0 & 5x – 2y – 7 = 0 & hence etc.] [Ans. (3, 4)]
14. Find the equation of the line passing through the intersection of the line 2x – 3y + 4 = 0 and
3x + 4y – 5 = 0 and is perpendicular to the line 6x – 7y + 8 = 0 [Ans. 119x + 102y = 125]
15. Find the co-ordinates of the foot of perpendicular from the point (2, 3) on the line x + y = 11.
[Ans. (5,6)]
16. Find the equation of a line passing through the point (4, –5) and perpendicular to a line passing
through (3, 4) and (6, 8). [ICWA June, 1986] [Ans. 3x + 4y + 8 = 0]
OBJECTIVE QUESTIONS :
1. Find the equation of straight line passing through the points and also state the gradient. ( –1, –3)
and (4, 6) [Ans. 9x – 5y – 6 = 0 ; 9/5]
2. Find the equation to the straight line c as its intercept on y axis and on as its gradient : c = –2,
m=2 3 [Ans. 3y – 2x + 6= 0]
3. Find the intercepts on the axes given line 3x – 5y = 9 [Ans. 3, –9/5]
4. Find the equations of the line passing through the point (1, 2) and having gradient as - 1 2 .

[Ans. x + 2y = 5 ]

5. If the slope of any straight line is 1 , then find the slope of a straight line which is parallel and
2
perpendicular to that straight line. [Ans. same slope for parallel;; for perpendicular it is –2]
6. Find the slope of the line parallel and perpendicular to line joining the points. (2, –1) and (0, 2).
[Ans. -3 2 ; 2 3 ]

7. x intercept of a line is 2 and it passes the point (1, 3); find she line. [ICWA (F) June, 2007]
[Ans. 3x + y + 6]
MATHS 4.23
1. Find the gradient of the line having equal intercepts on the axes of the first gradient of the co-
8.
ordinate system. [ICWA (F) Dec. 2006] [Ans. –1]
CO- ORIDINATE GEOMETRY

2. Find the equation of the straight line which passes through the points (3, 4) and making equal
9.
intercepts on the two axes. [ICWA (F) June, 2006] [Ans. x + y = 7]

10. Find the slope of the line perpendicular to the line joining the points (4, –3), (2, 1)
[ICWA (F) June, 2005] [Ans. 1 2 ]

11. Find the distance of the line x – 2y = 4 from the point (3, – 5) [Ans. 9 ]
5
12. Find the equations of a straight line passing through (– 2, 5) and parallel to x axis
[ICWA (F) June, 2004] [Ans. x + 2 = 0]
13. For what value of k. two lines 2x –y + 5 = 0 and 3y + kx + 1 = 0 are parallel? [Ans. –6]
14. For what value of k, two lines 2x + 5y + 1 = 0 and 5x + ky – 2 = 0 are perpendicular? [ Ans. – 2]

4.3 THE CIRCLE :


Introduction :
A circle is the locus of a point that moves keeping a fixed distance from a fixed point. The fixed point
is known as centre and the fixed distance is the radius.
FORMATION OF EQUATION :
1. When the centre is at origin and radius be equal to r.
XOX´, YOY´ are taken as co-ordinate axes.

Y Y

P
C Q

X
O N

X
O N N
Fig. 1
Fig. 2

Let P( x, y ) be any point on the circumference the circle, PN is drawn perpendicular to OX, Now ON
= x, NP = y, Op = r.
We know from the right-angled triangle OPN, ON2 + NP3 = OP2 (See Fig. 1)
Or, x2 + y2 = r2 which is the required equation.
4.24 MATHS
2. When the centre is at (h, k) and radius is r.
Let P(x, y) be any point on the circumference. We have CQ = x – h, PQ = y –k, CP = r (See Fig. 2)
From the right-angled triangle PCQ, we have CQ2 + PQ2 = CP2, or (x –h) +| (y –k)2 = r2. Which is the
equation of the circle.
Cor. If h = k = 0, then we get x2 + y2 = r2.

Note. (i) For the centre (– h, –k) and radius (r), equation of circle will be (x + h)2 + (y + k)2 = r2.
(ii) There are three unknowns, h, k and r in the equation of circle (x – h)2 + (y – k)2= r2.

EQUAION OF A CIRCLE IN FEW SPECIAL CASES :

C
X
O

(i) If the centre of the circle lies on x-axis, then k = 0


\(i)equation
If the centre of circle
of the the circle
willlies
be on
(x –x-axis,
h)2 + then
(y – k0)=2 =
0 r2
or, (x – h)2 + y2 = r2
\ equation of the circle will be (x – h) + (y – 0) = r or, (x – h)2 + y2 = r2
2 2 2
(ii) If the centre of the circle lies on y-axis, then h = 0. In that case, the equation of the circle will be
(ii) If the centre of the circle lies on y-axis, then h = 0. In that case, the equation of the circle will be
x2 + ( y –k)2 = r2.
x2 + ( y –k)2 = r2.
(iii) If the origin (0, 0) lies on the circumference of then circle, then N OC = r, co-ordinates centre (h,
(iii) If the origin (0, 0) lies on the circumference of then circle, the N OC = r, co-ordinates centre (h,
k) i.e., h2 2+ k2 2= r2.2
k) i.e., h + k = r .
Now, x2 2– 2xh + h22+ y22– 2yk + k22 = r22
Now, x – 2xh + h + y – 2yk + k = r
Or, x2 2+ y2 2– 2xh – 2ky = 0 (as h22 + k22 = r22)
Or, x + y – 2xh – 2ky = 0 (as h + k = r )
Note. In the equation of a circle passing through origin, three is no constant form.
Note. In the equation of a circle passing through origin, three is no constant form.
(iv) If the circle touches both axes, then h = k = r. Equation of the circle will be (x – 2r)2 + (y –2r)2 =2 r2.
(iv) If the circle touches both axes, then h = k = r. Equation of the circle will be (x – r) + (y – r) = r .
(v) Equation of a circle described on the line joining (x1, y1) and (x2, y2) as diameter. Let P(x , y) be a
(v) Equation of a circle described on the line joining (x1, y1) and (x2, y2) as diameter. Let P(x , y) be a
point on the circumference and A, B are the extremities of the diameter.
point on the circumference and A, B are the extremities of the diameter.
The
ThePA PAandandPB
PBareareatatright
right angles.
angles.
yy--yy1 y-y
Gradient
GradientofofPA
PA==x - x 1 and
and that
that of
of PB = y - y 22
PB =
x - x11 xx -- xx22
yy--yy1 yy -- yy2
By
Bycondition,
condition,we have x - x 1 .. x - x 2 =
wehave =-- 11 i.e.,
i.e., (x
(x –x
–x11))(x
(x––xx2)2)++(y(y––y1y)1)(y(y– –y2y)2=) =0.0.
x - x11 x - x 22

MATHS 4.25
GENERAL FORM :
The equation of a circle can be represented in the general form x2 + y2 + 2gx + 2fy + c = 0. Where
CO- ORIDINATE GEOMETRY

g, f and c are constants.


We know that the equation of a circle having centre at (h, k), and radius r is
(x – h)2 + (y – k)2 = r2 or, x2 + y2 – 2hx – 2ky + (h2 + k2 – r2) = 0

or, x3 + y2 + 2 (–h)x + 2(–k)y + (h2 + k2 – r2) = 0 or, r2 + y2 + 2gx + 2 fy + c = 0 taking


– h = g, – k = f, h2 + k2 – r2 = c.
Conversely, equation of the form x2 + y2 + 2gx + 2fy + c = 0 represents a circle, for the equation may
be written as x2 + 2gx + g2 + y2 + 2fy + f2 –g2 – f2 + c = 0
or, (x + g) 2 + (y + f)2 + g2 + f2 – c = 0
2
2
or, {x - (-g) 2 }+ {(y )- (-f )} = (g 2
)
+ f 2 - c ...........(a)

Which is the equation of the circle. Having

(i) Centre at (–g, –f). (ii) Radius g 2 + f 2 - c , provided g2 + f2 > c.

(a) Note. 1. If g2 + f2 – c = 0 or (g2 + f2 = c), the radius becomes zero, in which case the circle
reduces to a point (–g, –f), as only one point (– g, –f) satisfies eqn. (a). Such a circle is known as
point circle or ‘null circle’.
2. If g2 + f2 –c > 0, the radius is real.
3. If g2 + f2 –c < 0, the radius is imaginary and hence there is no circle.
Remember :
1. In the general from equation x2 + y2 + 2gx + 2fy + c = 0 if c = 0, the circle passes through the
origin i.e. (0, 0).
c = 0 means h2 + k2 – r2 = 0 or, h2 + k2 = r2 and hence the result.
2. If g = 0 the centre of the circle lies on y – axis
As g = 0 means – h = 0, (i.e.h = 0 ) and the equation reduces to
x2 + (y + f) 2 = f2 – c, i.e. x2 + y2 + 2fy+ c = 0.
3. If again f = 0, then the centre lies on x-axis.
As f = 0 reduces, k = 0, the eqn. reduces to (x + g) 2 + y2 = g2 – c
Or, x2 + y2 + 2gx + c =0.

CIRCLE THROUGH THREE GIVEN POINTS :


Let (x1 ,y1), (x2, y2), (x3, y3) be three points on the circle
x2 + y2 + 2gx + 2fy + c = 0.
Then we have x1 + y1+ 2gx1 + 2f y1 + c = 0 ……..(1)
2 2
x1 + y1 + 2gx2 + 2f y2 + c = 0 ……..(2)
2 2
x3 + y3 + 2gx3 + 2f y3 + c = 0 ……..(3)
4.26 MATHS
Solving these three equations, we can find the value of the constants g, f and c. Lastly putting these
three values in the general equation, we can find the required equation of the circle.
POINTS LYING OUTSIDE, ON OR INSIDE OF A CIRCLE :

Y P

X
O

A point P is said to lie outside, on or inside a centre C and radius r according as


>
CP r
<
Let x2 + y2 +2gx + 2fy + c = 0 be a circle, and P (x1,y1) be any point in the point in the plane. Now P
lies outside on or inside the circle according as
> > >
CP r, i.e. CP2 r2 , i.e., (x1 +g)2 + (y1+f)2 g 2 + r2 – c
< < <

>
or, x12 + y12+ 2gx1 + 2f y1 + c 0
<
obs : Two circles will touch externally or internally if the distance of the two centres is equal or less
than the sum of the radii.
SOLVED EXAMPLES :
1. Find the equation of the circle whose centre (2, –3) and radius 4.
Let the equation of the circle be (x –h)2 + (y – k)2 = r2, now h = 2, k = –3, r = 3
Putting these values, we find (x –2)2 + (y + 3)2 = 42
Or, x2 + y2 – 4x + 6y –3 = 0.
2. Find the co-ordinates of the centre and radius of the circle.
3x2 + 3y2 – 4x + 6 y – 4 = 0.

Dividing both sides by 3. The equation is x 2 + y 2 - 4 x + 2 y - 4 = 0


3 3

Where, g = - 2 , f = 1. c = - 4
3 3

\ centre æç 2 , - 1ö÷ , and radius


4 +1+ 4 = 5 .
è3 ø 9 3 3
3. Find the equation of the circle which passes through he points (1, 2), (3, 4) and (3, 2).
Let the equation of the circle be x2 + y2 +2gx + 2fy + c = 0.
MATHS 4.27
Since it passes through (1,2), (3, 4), and (3, 2) we have
1 + 4 + 2g + 4f + c = 0 or, 2g + 4f = c = – 5 …….(i)
CO- ORIDINATE GEOMETRY

9 +16 + 6g + 8f + c = 0 or, 6g + 8f + c = – 25 ……. (ii)


9 + 4+ 6g + 4f + c = 0 or, 6g + 4f + c = –13 …….(iii)
Subtracting eqn. (i) from eqn. (ii), we get 4g = – 8, g = – 2
Again subtracting (iii) from (ii),we find 4f = – 12, f = – 3.
From (I), 2 (– 2) + 44 (– 3) + c = – 5 or, c = 11
2 2
\ reqd. eqn. is x + y – 4x – 6y + 11 = 0
1.
4. Find the equation of the circle whose centre is (4, –5) and length of circumference is 8p units.
[ICWA (F) June, 2000]
Circumference of a circle = 2pr = 8p
or, r = 4 units,. Centre is at (4, – 5), so the equation of circle is (x – 4)2 + (y + 5)2 = 42
or, x2 + y2 – 8x + 10y + 25 = 0
5. Find the equation of the circle which is concentric with the circle x2 + y2 +12y – 8x + 15 = 0 and
which passes through the point (5, 4) [ICWA (F) Dec. 2000]
Here g = – 4, f =6, c = 15, so centre is at the point (4, – 6); and radius is the distance between the
centre (4, – 6) and the point (5, 4).

Now distance = (4 - 5) 2 + ( -6 - 4) 2 = 101 i.e. radius 101

\ Equation of the circle is (x – 4)2 + (y + 6)2 = 101.


6. Find the equation of the circle whose centre is the point (2, 3) and which passes through the point
(5, 7).
From (x – h)2 + ( y – k)2 = r2, we have

Radius (r) = ( x - h ) 2 + ( y - k ) 2 = (5 - 2) 2 + (7 - 3)2

Equation of the circle (x –2)2 + (y – 3)2 = 52


Or, x2 + y2 – 4x – 6y –12 = 0.
Alternatively, equation of the circle is (x – 2)2 + (y – 3)2 = r2 as it has centre (2, 3). Now the circle
passes through (5, 7); so we have
(5 –2)2 + (7 –3)2 = r2 or, r2 = 25 = 52 \r = 5.
\ the equation of the circle is (x – 2)2 + (7 – 3)2 = 52
or, x2 + y2 – 4x – 6y – 12 = 0
7. Find the equation of the circle on the join of (2, 3) and (4, 5) as diameter.
Here, x1 = 2, y1 = 3, x2 = 4, y2 = 5
\ the equation is (x – 2) (x – 4) + (y –3) (y – 5) = 0 i.e. x2 + y2 – 6x –8y + 23 = 0
8. Does the point (1, 2) lie outside the circle x2 + y2 +16x + 14y –8 = 0.
Now putting the values of x, y in left side of the eqn. we find 12 + 22 + 16.1+ 14.2 –8 = 41>0. So the
point (1, 2) lies outside the circle.
4.28 MATHS
9. Find the equation to the circle which passes through the point (3, 4) (– 1, 6) and has its centre on
the straight line 3x + 5y – 2 = 0
Let (h, k) be the centre of the circle, and radius be a. Now the distances of the two given points from
the centre are same and equal to a.
(h – 3)2 + (k – 4)2 = (h + 1)2 + (k – 6)2 = a2 ……(i)
2 2 2 2
or, h – 6h + 9 + k – 8k + 16 = h + 2h + 1 + k – 12 k + 36 or, – 2h + k = 3
Again (h, k) lies on the line 3x + 5y–2 = 0 ………(ii)
So 3h + 5k –5 = 0
Solving (ii) and (iii), we find h = –1, k = 1 .…….(iii)
2 2 2 2
From (I), (–1–3) + (1 – 4) = a or, a = 16 + 9 = 25
The equation of the circle becomes (x + 1)2 + (y – 1)2 = 25 or, x2 + y2 + 2x – 2y – 23 = 0.
10. Show that the two circles x2 + y2 + 6x +14y +9 = 0, and x2 + y2 – 4x – 10y – 7 = 0 touch each
other externally and also determine the co-ordinates of point of contact. [ICWA (F) Dec. 2005]
Two circles will touch externally if sum of the radii of two circles is equal to the distance between the
two centres.
For 1 st circle : g = 3, f = 7, c = 9 so centre is at ( – 3, –7)

and radius ( r1 ) = g 2 + f 2 - c = 9 + 49 - 9 = 7

For 2nd cirlce : g = –2, f = –7, centre is at (2, 5)

and radius ( r2 ) = g 2 + f 2 - c = 4 + 25 + 7 = 6

\r1 + r2 = 6 + 7 = 13
again distance the centeres (– 3, – 7) and (2, 5)

= (2 + 3)2 + (5 + 7 )2 = 25 + 144 = 169 = 13 = r1 + r2 , hence the result.

m.2 + n.( -3) 7.2 + 6.( -3) - 4


For point of contact : x = = = (as m = 7, n = 6 if ratio is m : n)
m+n 7+6 13
7.5 + 5.( -7)
and y = =0
13

Reqd. co-ordinates are æç - 4 , 0 ö÷


è 13 ø
11. Find the equation of the circle whose centre lies on x-axis and which passes through (0, 4) and
(3, –1).

Since the centre lies on x-axis, co-ordinates of centre are (–g, 0). Radius is g2 - c .

Distance of any point (x, y) on the circumference of the circle is equal to radius of circle.
So we get, (x + g)2 + y2 = g2 – c
Since the circle passes through (0, 4) we get
0 + g 2 + 42 = g2 – c or, 16 = – c or, c = – 16
2 2 2
Again, (3 + g) + ( –1) = g – c or, 9 + 6g + g +1 = g2 + 16
2

MATHS 4.29
or, 6g = 6 or, g =1
\ reqd. eqn. is (x + 1)2 + y2 = 1 – (–16), or x2 + y2 + 2x – 16 = 0
CO- ORIDINATE GEOMETRY

SELF EXAMINATION QUESTIONS :


1. Find the equation of the circle which is concentric with the circle x2 + y2 + 6x + 7 = 0 and
passes through the point (1,1) [Ans. x2 + y2 + 6x – 8 = 0]
2. Find the equation of the circle which is concentric with the circles x2 + y2 – 4x + 6y – 3 = 0 and
which passes through the point (5, –2). [Ans. x2 + y2 – 4x + 6y + 3 = 0]
3. Find the equation of the circle concentric with the circle x2 + y2 + 4x – 6y – 13 = 0 and passing
through the centre of the circle x2 + y2 – 8x – 10y – 8 = 0. [Ans. x2 + y2 + 4x – 6y = 27]
4. Find the equation of the circle which passes through the points :
(i) (0, 2), (2, 0) and (2, 2)
[Ans. x2 + y2 – 2x – 2y = 0]
(ii) (1, 1) (5, – 5) (6, – 4) [Ans. x2 + y2 – 6x + 4y = 0]
5. Find the equation of the circle whose centre is at the origin and which meets the straight line
x - y = 1 on the axis of y. [Ans. . x2 + y2 = 36]
5 6
6. Find the radius of the circle whose centre is the point (1, –2) and which passes through the point
of intersection of the straight lines 3x + y = 14 and 2x + 5y = 18. [Ans. 5]
7. Find the equation of the joining the centres of the two circles . x2 + y2 – 2x + 4y – 1 = 0; x2 + y2
+ 2x – 4y + 1 = 0. (C.A. Inter. May 1976) [Ans. 2x + y = 0]
8. Find the equation of the circle which touches the co-ordinate axes at (1, 0) and (0,1),
[Ans. x2 + y2 – 2x – 2y + 1 = 0]
9. Find the equation of the circle which through the origin and cuts off intercepts 5 and 7 from the
positive directions of the axes of x and y. [Ans. x2 + y2 – 5x – 7y = 0]
10. Find the equation to the circle which passes through the points (4, 1) and (6, 5) and has its centre
on the line 4x + y = 16 [ICWA(F) June, ‘04]
2 2
[Ans. x + y – 6x – 8y + 15 = 0]
11. A circle has its centre on the line 2x – 3y + ! = 0 and passes through the points (4, 5) and (–2, –3) .
Find its equation. [Ans. x2 + y2 – 2x – 2y – 23 = 0 ]
12. Prove that the centres of the three circles
x2 + y2 = 1, x2 + y2 + 6x – 2y –1 = 0,
x2 + y2 – 12x + 4y – 1 = 0 lie on a right line.
13. Prove that the two circles x2 + y2 + 4x + 10y – 20 = 0 and x2 + y2 – 8x + 6y + 16 = 0 touch each
other externally and also determine the co-ordinates of point of contact.
[ICWA (F) Dec. 2006][Ans. 11/3, 3/5]
2 2
14.
4.
1. Show that the point (1, –2) lies outside the circle x + y – 8x + 6y + 16 = 0.

4.30 MATHS
15.
5.
1. Find the equation of the circle whose centre lies on the y-axis and which is passing through
(– 2, 2) and (4, 0). [Ans. x2 + y2 + 4y – 16 = 0]
16.
6.
2. Find the equation of the circle having its centre at x-axis and passing through (0,3) and (4, 1)
[ICWA (F) June 1998] [Ans. x2 + y2 – 2x – 9 = 0]
17.
7.
3. Find the equation of a circle such that :
It touches both the axes in the first quadrant at a distance of 4 units from the origin.
[Ans. x2 + y2 – 8x – 8y + 16 = 0]
18. Which statements is correct and why? “The point (2, 1) lies on, inside or outside the circle
x2 + y2 – 4x – 6y + 9 = 0”. [Ans. ‘On’]
19. Prove that :
x2 + y2 – 2x – 2y – 2 = 0 and x2 + y2 – 10x + 4y + 20 = 0. touch each other externally.
20. Prove that
x2 + y2 – 12x – 28 = 0 and x2 + y2 – 6x + 8y + 16 = 0 touch each other internally.

OBJECTIVE QUESTIONS :
1. Find the co-ordinates of centre of the circle x2 + y2 – 4x + 8y + 4 = 0. Find also its radius.
[Ans. (2, 4), 4 units]
2 2
2. Find the diameter of the circle x + y – 8x + 10y – 8 = 0. [Ans. 14 units]
3. The co-ordinates of the extreme points of a circle are (5, 4), (– 1, –6). Find the equation of the
circle. [Ans. x2 + y2 – 4x + 2y – 29 = 0]
4. A circle touches both the axes at a distance + 3 units from the origin. Find the equation of the
circle. [Ans. x2 + y2 – 6x – 6y + 9 = 0]
5. The diameter of the circle x2 + y2 + 4x – 7y – k = 0 is 9; find k. [Ans. 4]
2 2
6. Find the centre and radius of the circle 3x + 3y – 36x + 12y + 12 = 0 [hints : divide by 3 & etc.]
[Ans. (4, –2); 4 units]
7. The co-ordinates of the centre of the circle are (–5, –6), diameter 20; find the equation of the
circle. [Ans. x2 + y2 + 10x + 12y – 39 = 0]
8. Find the equation of the circle whose centre is at (–2, –3) and passes through (1, –1)
[Ans. x2 + y2 + 2x + 6y – 12 = 0]
9. Find the length of intercept by the circle x2 + y2 – 6x – 14y + 33 = 0] on y-axis. [hints : Put x = 0,
and y2 – 14y + 33 = 0 & etc.] [Ans. 8 units]
10. The diameter of a circle has the extreme points (4, –6) and (– 2, 2) respectively. What would be

the centre and radius of the circle? [Ans. (1, –4); 13 units]
11. If the centre of a circle is (3, 7) and its diameter is 10 units, then find the equation of the circle.
[Ans. x2 + y2 – 6x – 14y + 33 = 0]
12. The two diameter ends of a circle are (3,7) and (5, 2), find the equation of the circle and its centre.
[Ans. x2 + y2 – 8x – 9y + 29 = 0 ; (4, 9 2 )]
13. Find whether the point ( –5, 12) lies inside or outside the circle x2 + y2 =144. [Ans. outside]
2 2
14.
1. Show that the point (1, –2) lies outside the circle x + y – 8x + 6y + 16 = 0
MATHS 4.31
3.4 PARABOLA :
Definition : A parabola, is the locus of a point which moves in a plane such that its distance from a
fixed point (known as focus) is always equal to its perpendicular distance from a fixed straight line
(known as directrix), i.e. SP = PM (Sec fig.)

Standard Equation of Parabola :

Y
M
P(x,y)
M
Directrix

(a,2a)
L
Z
S (a,0) X
A N
(0,0)
x+a=0

L
(a,–2a) y 2=
4ax
M´ P´

Let S be focus of the parabola, MM´ be the directrix.


Let XZ be divided at A. i.e., ZA = AS = a, A will be vertex, of the parabola, i.e., A is origin (0, 0).
ZAS is extended upto X. AY is drawn perpendicular to ZAS. Now AX and AY are taken x and y axes.
If the co-ordinates of S be (a, o), then the equation of directrix MM´ will be x = – a or x + a = 0.
Let P (x, y) be any point on the curve. Join SP.
Draw PN and PM perpendiculars on x-axis and MM´.

BY definition. SP = PM or SP2 = PM2, SP = (x - a )2 + (y - 0)2 = (x - a )2 + y 2


As PM is perpendicular to x + a = 0 (Eqn. of directrix), so PM = x + a = x + a
12 + 0

From (i), = (x - a )2 + y 2 = x + a or (x –a)2 + y2 = (x + a)2


or, x2 – 2ax + a2 + y2 = x2 + 2ax + a2
or, y2 = 4ax, (a > 0) which is the required equation.

4.32 MATHS
Note. Here co-ordinates of focus S and vertex A are respectively (a, 0) and (0, 0); equation of directrix
is x + a =0, eqn. of axis is y = 0, eqn. of tangent at A (0, 0) is x = 0. This tangent is at perpendicular to
x-axis.
The equation of y2 = 4ax is not true for x < 0. The parabola is symmetrical about x-axis.
LATUS RECTUM :
Let the chord LL´ be perpendicular to x-axis. L and L´ lies on the curve. LL´ is the latus rectum of the
parabola. Draw LK perpendicular to MM´.
Now LL´ = 2.LS = 2. KL = 2.2 AS = 4 AS = 4a. (a >0)
i.e., length of the latus rectum = 4a units.
Again SZ = 2. AS = 2a i.e., distance of S from directrix is 2a units.
Note. If latus rectum is known, the equation of parabola can be determined.

Few Definitions :
(i) Axis. The straight line (AX) drawn through the focus perpendicular to the directrix is
known as axis of the parabola. In this axis y = 0 ; so the equation of the axis is y = 0.
(ii) Vertex. The point (A) where the axis of the parabola meets this curve is called the vertex.
Its co-ordinates are (0, 0).
(iii) Focal distance. The distance (SP) of any point (P) on the curve from its focus is known as
focal distance.
Let P (x, y) be any point on the curve, its distance upto S (a, 0) is
SP = PM = ZN = AN = a + x.
(iv) Double ordinate. A straight line, drawn perpendicular to the axis of the parabola and
bisected by the axis is called double ordinate. Here PNP¢ is the double ordinate of P.
(v) Latus rectum. The double ordinate (LSL¢) passing through the focus (S) is known as latus
rectum. Its length is 4a (a > 0).
Note : Two parabolas will be equal to each other if their latus rectums are equal.
(vi) Directrix. It is parallel to y-axis and passes through Z (– a, 0). So its equation is x = – a or
x + a = 0.
Remember :
In case of a parabola y2 = 4ax
(i) co-ordinates of vertex : (0, 0), (ii) co-ordinates of focus : (a, 0)
(iii) equation of directrix : x + a = 0, (iv) equation of axis : y = 0.
(v) length of latus rectum : 4a, (vi) end-points of latus rectum : L, L¢ ; their co-ords ;
(a, 2a), a, – 2a)
(vii) co-ordinates of Z (on directrix) ; (–a, 0), (viii) equation of tangent at vertex is x = 0.

MATHS 4.33
Y
CO- ORIDINATE GEOMETRY

x
2
=4
P (x,y)

ay
S
(0,a)

X´ X
A(0,0)

Directrix y+a=0 Z M

S(–a,0)
X´ X
A
Directrix x–a=0

– 4ax
y =
2


Y

Directrix y–a=0
A(0,0)
X´ X

S(0,–a)
y
4a
=–
x2

4.34 MATHS
3. If the vertex of the parabola be the origin and the axis of the curve be y-axis and also the focus is
below the x-axis, then the equation of the parabola is
x2 = – 4ay, (a, > 0)
Hence the co-ordinates of the focus are (0, – a) and those of vertex (0, 0). The equation of directrix
is y = a and that of axis is x = 0.
Latus rectum is 4a, (a > 0).
Asix parallel to x or y axis :
Let the vertex A be (h, k) and the latus rectum be 4a. Draw AX¢, AY¢ parallel to the axes.
1. Let the axis of the parabola be parallel to x-axis, i.e., along AX¢. Now with reference to AX¢ and
AY¢, the equation of the parabola is y2 = – 4ax.


Y
P
M

X´ Z A a N X´
S

h k

O X


If now the origin be transferred from A to O, then we get the equation referred to OX and OY as axes.
Then point A being (h, k), the co-ordinates of O referred to AX¢ and AY¢ as axes are (– h, –k). Hence
the transformed equation is (y – k)2 = 4a (x – h) which is the required equation to the parabola whose
vertex is (h, k) and whose axis is parallel to x-axis.
2. If the axis of the parabola be parallel to y-axis, then similarly putting x – h and y – k for x and y
respectively. In the Eqn.. x2 = 4ay, we get (x – h)2 = 4a (y – k) as required equation.
Summary :
a = distance between focus and vertex, in every case (say).
Parabola y2 = 4ax x2 = 4ay y2 = – 4ax x2 = – 4ay (y – k)2 (x – y)2
Items = 4a (x – h) = 4a (y – k)
1. co-ords. of focus a, 0 0, a – a, 0 0, – a a + h, k h, a+k
2. co-ords. of vertex 0, 0 0, 0 0, 0 0, 0 h, k h, k
3. axis x, + ve y, + ve x, – ve y, – ve parallel to x parallel to y
4. eqn. of axis y=0 x=0 y=0 x=0 x=h y=k
5. length of latus 4a 4a 4a 4a 4a 4a
rectum
6. equ. of directrix x+a=0 y+a=0 x–a=0 y–a=0 x+a=h y+a=k

MATHS 4.35
rectum
6. equ. of directrix x+a=0 y+a=0 x–a=0 y–a=0 x+a=h y+a=k

To find the eqn. of parabola if coordinates of the focus and eqn. of directrix are known :
CO- ORIDINATE GEOMETRY

Let the co-ordinates of focus (S) be (h, k) and equation of directrix be ax + by + c = 0. P (x, y) be any
point on the curve. Now SP = PM (PM is perpendicular distance from P on directrix).

SP = (x - h )2 + (y - k )2
ax + by + c
PM = (by formula of perpendicular. distance)
a 2 + b2

Since SP2 = PM2, so (x – h)2 + (y – k)2 =


(ax + by + c ) 2

a 2 + b2
or (a2 + b2) { (x – h)2 + (y – k)2 } = (ax + by + c2) which is the required equation.
Example : Given focus S (0, 0), directrix 3x + 4 = 0. Find the equation of parabola and also the focal
distance.
Let P(x, y) be any point on the curve. The perpendicular distance from P on directrix.

=
3y + 4
, again SP = x 2 + y 2 So,
(3y + 4 )2 = x 2 + y2
9 9
or, 9x2 = 24y + 16 which is the reqd. eqn.
3 .0 + 4 4
Again the perpendicular distance from S (0, 0) on directrix, 3y + 4 = 0 is = .
9 3

4 8
Focal distance = 2 ´ =
3 3
Any straight line y = mx + c cuts parabola y2 = 4ax in two points.
Eliminating y, we find, (mx + c)2 = 4ax or m2x2 (2mc – 4a)x + c2 = 0 …(I)
which is a quadratic equation and gives two values of x (real, coincident or imaginary).
To find the condition that the line y = mx + c touches the parabola y2 = 4ax, and hence to find the
co-ordinates of point of contact.
The line will be a tangent to the parabola of Eqn. (1) (see above) has two roots equal and the
corresponding condition is,
4m2c2 = (2mc – 4a)2 (discriminant is zero)
or 4m2c2 = 4m2c2 – 16amc + 16a2 or 16mc = 16a2
a
or c =
m
a
So, for any value of m, the line y = mx + is a tangent to the parabola y2 = 4ax.
m
Position of a point in relation to a Parabola :

4.36 MATHS
Position of a point in relation to a Parabola :
A point P(x1, y1) will lie outside, on or inside a parabola y2 = 4ax if y12 > = < 4ax1
or, if y12 – 4ax1 > = < 0.
Example : Examine whether the points : (i) (3, 2) ; (ii) (2, 4) lie inside or on the parabola y2 = 6x.
(i) Here x1 = 3, y1 = 2. Putting the values of x1 and y1 in y2 – 6x, we find 22 – 6.3 = 4 – 18 =
–14 < 0. So the point (3, 2) lies inside the curve.
(ii) Similarly, 42 – 6.2 = 16 – 12 = 4 > 0. So the point lies outside the curve.
SOLVED EXAMPLES :
Example 1. For every parabola given below, find vertex, axis, focus, latus rectum, directrix and
equation of tangent at the vertex :
(i) 3y2 = 4x.
(ii) x2 – 6x – 3y – 3 = 0 [ICWA (F) June 2006]
4 4 1
(iii) From 3y2 = 4x, we get y 2 = x. Comparing this with y2 = 4ax, we find 4a = or a = .
3 3 2
Vertex : (0, 0). Axis : x-axis, + ve, i.e., y = 0.
æ1 ö 1 4
Focus : (a, 0) i.e., ç , 0 ÷ . Latus rectum : 4a = 4. = .
è3 ø 3 3

1 1
Directrix : x = – a, i.e., x = - or x + = 0 or 3x + 1 = 0.
3 3
Tangent at vertex : x = 0
(ii) x2 – 6x – 3y – 3 = 0 or, x2 – 6x + 9 = 3y + 3 + 9
3
or (x – 3)2 = 3 (y + 4) or, X2 = 3Y or X2 = 4. Y
4
where X = x – 3 and Y = y + 4.
Vertex : X = 0, Y = 0, i.e., x– 3 = 0, y + 4 = 0 i.e., (3, – 4)
Axis : X = 0, i.e., x – 3 = 0 (parallel to y-axis)
3 3 æ 13 ö
Focus : X = 0, Y = or, x – 3 = 0, y + 4 = or, ç 3,- ÷ .
4 4 è 4ø
Latus rectum : 3. Tangent at vertex : y = – 4, i.e., y + 4 = 0.
3
Directrix : Y + =0
4
3
or y + 4 + =0
4
or 4y + 19 = 0.
2. For the equation of parabola y2 – 6y – 12x – 3 = 0, find the focus, directrix and the length of latus
rectum. [ICWA (F) June 2005]
y2 – 6y – 12x – 3 = 0 or, y2 – 6y + 9 = 12x + 3 + 9
or, (y – 3)2 = 12 (x + 1) or, (y – 3)2 = 4. 3 {x – ( – 1)}2.

MATHS 4.37
Comparing with (y – k)2 = 4a (x – h) we get, h = – 1, k = 3, a = 3.
Focus : (a + h, k) or, (3 – 1, 3) or, (2, 3)
CO- ORIDINATE GEOMETRY

Directrix : x + a = h or, x + 3 = – 1 or, x + 4 = 0


Length of rectum : 4a = 4. 3 = 12 units.
3. Find the co-ordinates of vertex and focus and also the length of latus rectum of the following
parabola
parabolas :
y2 y= 4(x
2
(i) 4(x++y)y) [ICWA (F) Dec. ’2000] ;
2 2
y = 4(x + y) or y – 4y = 4x
or, (y – 2)2 = 4(x + 1)
or, Y2 = 4X, where Y = y – 2, X = x + 1.
Vertex : X = 0, Y = 0,
(i.e., x + 1 = 0, y – 2 = 0, i.e., (– 1, 2).
Focus : X = a, Y = 0,
i.e., x + 1 = 1 (as a = 1), y – 2 = 0. i. e., (0, 2)
Latus rectum : 4 units.
(Also Directrix : X + A = 0 or x + 1 + 1 = 0 or x + 2 = 0. Axis : y – 2 = 0)
4. Parabola y2 = px passes through (4, – 2). Find the latus rectum and focus.
Since y2 = px passes throug (4, – 2), we find (– 2)2 = p.4
or, p = 1. So the equation of parabola is y2 = x.
1 1 1 æ1 ö
or, y2 = 4. .x, as, a = . So latus rectum = 4a = 4. = 1, focus (a, 0) or ç , 0÷.
4 4 4 è4 ø
EQUATION OF PARABOLA UNDER DIFFERENT GIVEN CONDITIONS :
Vertex and focus given :
5. If the co-ordinates of vertex and focus of a parabola are respectively (4, 6) and (7, 6) then find the
equation of the parabola.
Since the ordinates (i.e., y-co-ordinates) of vertex and focus are same, the axis of the parabola is
parallel to x-axis.
A = distance of vertex and focus = abscissa of focus – abscissa of vertex = 7 – 4 = 3.
\ the equation is (y – k)2 = 4a (x – h)
or (y – 6)2 = 4.3 (x – 4), as h = 4, k = 6, a = 3
or y2 – 12y + 3y = 12x – 48
or y2 – 12y – 12x + 84 = 0.
Note :
If y-co-ordinates, i.e., ordinates of vertex and focus be same, then the problem can be solved by the
above process. If, however, the ordinates are not same, then such problem can be solved by the
following method.

4.38 MATHS
Focus and Directrix given :
6. Find the equation of a parabola whose focus is (– 1, 1) and equation of directrix is x + y + 1 = 0.
Find also the length of latus rectum and the equation of axis. [ICWA (F) Dec. 2004]
Let P(x, y) be any point on the curve. Since distance of P from focus and its perpendicular distance
from directrix is same, we get
x + y +1
(x + 1)2 + (y - 1)2 =
1+1
2
or (x + 1)2 + (y - 1)2 = (x + y + 1)
2
or 2(x + 2x + 1 + y – 2y + 1) = x2 + y2 + 1 + 2xy + 2x + 2y
2 2

or x2 + y2 + 2x – 6y – 2xy + 3 = 0.
Which is the reqd. eqn.
Perpendicular distance from focus S (–1, 1) on directrix
-1 + 1 + 1 1 1
x + y + 1 = 0 is = \ Latus rectum = 2. = 2 units.
2 2 2 2
1 +1
The equation of straight line passing through focus (– 1, 1) and perpendicular to the directrix
(x + y + 1) = 0 is the reqd. equation of axis. From x + y + 1 = 0, we get y = – x – 1, i.e., slope
(m1) = – 1. Any line passing (– 1, 1) is y – 1 = m (x + 1). Now
m. m1 = – 1 or m. (– 1) = – 1 or m = 1
\ reqd. equation is y – 1 = x + 1 or x – y + 2 = 0.
Vertex and directrix given :
7. Find the equation of the parabola whose vertex is at (1, 2) and whose directrix is the line y + 1 = 0.
Directrix is y + 1 = 0 or y = – 1, i.e., parallel to x-axis. So the axis of the parabola will be parallel to
y-axis. Again vertex is (1, 2) i.e., above the x-axis (or the directrix). The parabola is open upward.
Equation of the parabola will be in the form of (x – h)2 = 4a (y – k)
Here h = 1, k = 2, a = distance between the vertex and directrix, i.e., 3.
\ Reqd. Eqn. is (x – 1)2 = 4.3 (y – 2)
or x2 – 2x – 12y + 25 = 0.
Passing through three given points :
8. Find the equation of the parabola whose axis is parallel to the y-axis and which passes through the
points (0, 2), (1, 6) and (– 2, 12).
The axis of the parabola is parallel to y-axis, so the equation may be written as
Y = ax2 + bx + c …(i)
Where a, b, c are constants.
Since the parabola passes through the points (0, 2), (1, 6) and (– 2, 12) we find from (1)
2 = 0 + 0 + c or c = 2 …(ii)
6 = a + b + c = a + b + 2 or a + b = 4 … (iii)
12 = 4a – 2b + c = 4a – 2b + 2 or 4a – 2b = 10 …(iv)
MATHS 4.39
Solving (iii), (iv), we get a = 3, b = 1
The reqd. eqn. is y = 3x2 + x + 2.
CO- ORIDINATE GEOMETRY

SELF EXAMINATION QEUSTIONS :


1. Find the co-ordinates of vertex, focus, equation of directrix, axis and length of latus rectum of the
following parabolas :
(i) y2 = 8x, (ii) 5y2 = 7x, (iii) x2 = – 20y, (iv) 2x2 + 3y = 0.
æ 7 ö 7
[Ans. (i) (0, 0) ; x + 2 = 0, y = 0 ; 8 (ii) (0, 0) ; ç , 0 ÷ ; 2x + 7 = 0 ; y = 0 ;
è 20 ø 5
(iv) (0, 0) ; (0, – 5) y – 5 = 0 ; x = 0 ; 20
æ 3ö 3
(v) (0, 0) ; ç 0, - ÷ ; 8y – 3 = 0, x = 0 ; ]
è 8ø 2
2. Find the equation of the parabola whose vertex is (0, 0) and also satisfies the following conditions :
(i) focus at (3, 0), (ii) focus at (– 2, 0)
(iii) directrix x = 5, [Ans. (i) y2 = 12x, (ii) y2 = – 8x, (iii) y2 = 20x,]
3. Find the equation of the following parabola :
(i) focus at (3, 0), directrix x = – 3. [Ans. y2 = 12x]
(ii) focus at (0, 6), directrix y = 0 [Ans. x2 = 12 (y – 3)]
(iii) focus at (2, 1), directrix 3x – 2y + 1 = 0. [Ans. x2 + 9y2 – 46x – 18y – 6xy + 49 = 0]
4. Find the equation to the parabola : whose focus is at the origin and whose directrix is the straight
2
line 2x + y – 1 = 0. [Ans. x2 + 4y2 – 4xy + 4x + 2y – 1 = 0 ; 5]
5
5. Find the vertex, axis, directrix and the latus rectum of the following parabolas :
(i) y2 – 2y + 8x – 23 = 0
(ii) x2 + 8x + 12y + 4 = 0 [Ans. (i) (3, 1) ; (1, 1) ; y = 1 ; x = 5 ; 8
(i) (– 4, 1) ; (– 4, – 2) ; x + 4 = 0 ; y = 4 ; 12]
2
6. In the parabola 4(y – 1) = – 7(x –3), find :
(i) latus rectum
7 41
(ii) co-ordinates of focus and vertex. [Ans. (i) ; (iii) , 1, (3, 1)]
4 16
7. Parabola y2 = 4 px passes through (3, – 2). Find the latus rectum, co-ordinates of focus of the
4 æ1 ö
parabola. [Ans. , ç , 1÷ ]
3 è3 ø
8. Find the equation of a parabola whose vertex is (0, 0) and passes through (4, 5) and whose
(i) axis is y = 0, (ii) axis is x = 0. [Ans. (i) 4y2 = 25x, (ii) 5x2 = 16y]
9. Find the equation of a parabola whose vertex is at the origin and axis coincides with x-axis and
latus rectum is 8.
[Hints. Axis is to be taken both on + ve and – ve sides of x-axis] [Ans. y3 = ± 8x]
10. Find the equation of a parabola, such that :
4.40 MATHS
(i) Vertex is origin and focus is (4, 0).
(ii) Vertex is (– 2, 2) and focus is (1, 2). [Ans. (i) y2 = 16x (ii) (y – 2)2 = 12 (x + 2)]
11. Find the equation of the parabola having : Vertex at (2, 1) and directrix y + 2 = 0
[Ans. x2 – 4x – 12y + 16 = 0]
12. The focus of a parabola is (– 4, 0) and its vertex is (– 2, 2) ; find its equation.
[Ans. x2 + y2 – 2xy + 24x + 8y + 16 = 0]
13. Find a point on the parabola y2 = 8x, whose focal distance is 4. If again the focal distance of any

point on the curve is 8, find the co-ordinate of that point. (


[Ans. (2, ± 4 ) ; 6, ± 4 3 ] )
14. (a) Examine whether the points (i) (– 4, 1), (ii) (2, 4), (iii) (4, 3) lies inside, outside or on the
parabola y2 = 8x. [Ans. (i) outside, (ii) on, (iii) inside.]
(b) Show that the point (– 4, 3) lies outside but the point (4, – 3) lies inside the parabola y2 = 4x.
OBJECTIVE QUESTIONS :
1. Find the coordinates of (i) focus, (ii) vertex, (iii) equations of directrix, (iv) equation of axis,
(v) length of latus rectum of the parabola y2 = 4x.
[Ans. (i) (1, 0) (ii) (0, 0) (iii) x + 1 = 0, (iv) y = 0 (v) 4 units.]
2. Find the equation of directrix and coordinates of focus of the parabola y2 = 8x
[Ans. x + 2 = 0, (2, 0)]
2
3. Find the latus rectum and coordinates of focus of the parabola y = 12x [Ans. 12 units ; (3, 0)]
4. Find the equation of parabola having focus at (3, 0) and directrix x + 3 = 0 [Ans. y2 = 12x]
5. Find the equation of parabola whose vertex is at (0, 0) and focus (– 2, 0) [Ans. y2 = – 8x]
6. Find the vertex and focus of the parabola y2 – 2y + 8x – 23 = 0 [Ans. (3, 1) ; (1, 1)]
2
7. If the parabola x = ay passes through the point (6, – 2), find the length of the latus rectum.
[Ans. 72 units]
2
8. If the parabola x = ay passes through the point (1, 2) ; find the equation of the directrix.
[Ans. 2y + 1 = 0]
9. Find the point on the parabola y2 = 12x at which the ordinate is double the abscissa. [Ans. (3, 6)]
2
10. Find the coordinates of focus and length of latus rectum of the parable 5y = 36x.

( )
[Ans. 9 , 0 , 36 units]
5 5
11. Find the focus and length of latus rectum of the parabola y2 = 4ax which passes through the point

(– 1, 3) (
[ICWA (F) June 2007] [Ans. - 9 , 0 ; 9 units.]
4
)
12. Find the vertex and length of latus rectum of the parabola (y + 3)2 = 2 (x + 2)
[ICWA (F) June 2006] [Ans. (– 2, – 3) ; 2 units]
2
13. Find the vertex of the parabola y – 2y – 8x = 23 [ICWA (F) Dec. 2005] [Ans. (– 3, 1)]
14. Find the equation of the parabola whose vertex is (0, 0) and directrix is y = 2 [Ans. x2 = 8y]
4.5 ELLIPSE :

MATHS 4.41
Y
L L
CO- ORIDINATE GEOMETRY

B (0,b) P(x,y)
M´ M

A´ A(a,0) Z
X´ X
(–a,0) S(–ae,0) C(0,0) N S(ae,0)
x+ ae = 0

x– ae = 0
B´(0,–b)


Definition :
An ellipse is the locus of a point that moves in a plane in such a way that its distance from a fixed
point (S) in the plane always bears a constant ratio e (0 < e < 1) with its perpendicular distance from a
fixed straight line (L) of the plane, not passing through S.
The fixed point (S) is called focus and the fixed line (L) is the directrix, and e is known as eccentricity
of the ellipse.
x 2 y2
Standard equation is + = 1. This is true for any point P on the ellipse.
a 2 b2
Deductions :
a 2 - b2
(
1. Here b 2 = a 2 1 - e 2 or e 2 = ) a2
, a positive quantity. So a > b or 2a > 2b i.e., major axis >

minor axis. We refer AA´ is major axis. BB´ as minor axis.


2. If b > a, then the major axis will be in the direction of y a axis, and minor axis along x-axis. In
x 2 y2
that case ellipse would be + =1
b2 a 2
3. Equation of major axis is y = 0, that if minor axis is x = 0.
Few terms :
(Refer the first figure)
Vertices : The points A and A´ where the lines joining the foci meets the curve (i.e. the ellipse) are
called the vertices of the ellipse.
Centre : The middle point of AA´ (i.e., C) is called the centre of the ellipse.
Axes : The line AA´ is called the major axis and the line BB´ is called the minor axis.
Latus Rectum : The double ordinate passing through the focus is called the latus rectum of the
ellipse.
[In the fig. LSL¢ is the latus rectum passing through the focus S.]
4.42 MATHS
Latus Rectum :

P
L

X¢ X
C N S


If from P (x, y), PN is drawn perpendicular to the major axis and produced to meet the curve at P´ then
PN = ordinate of P.
PNP´ = double ordinate of P.
Now a double ordinate passing through focus (S) is known as latus rectum of the ellipse. There are
two latus recta of the ellipse.
Co-ordinates of S are (ae, 0), and those of L are (ae, SL), since, L lies on the ellipse x2/a2 + y2/b2 = 1.
We have
a 2 e 2 SL2 SL2
+ = 1 or , = 1 - e2
a2 b2 b2

b2 b4
( )
or SL2 = b 2 1 - e 2 = b 2 =
a2 a2

b2
\ SL =
a
2b 2
and LSL¢ (i.e. latus rectum) = LS + SL¢ = 2SL = .
a
Note.
(i) Length of semi-latus rectum = b2/a.
(ii) For x2 / a2 + y2 / b2 = 1 and x2 / b2 + y2/ a2 = 1, latus rectum (in each case) is b2 / a.
1 2
The eccentricity : From the relation b2 = a2 (1 – e2), we get eccentricity i.e., e = a - b 2 , where,
a
a = semi-major axis. b = semi-minor axis.
Corr.
(i) Equation of latus rectum through S is x = ae and through S´ that eqn. is x = + ae.
(ii) Co-ordinates of L (LSL¢ through S) are (ae, b2 / a) and (– ae, – b2 / a).
(iii) Equation of MZ is x = ae or, x – a/e = 0 ; equation of M¢ Z¢ is x = – a/e.
(iv) Equation of LSL¢ is x = ae or x – a/e = 0 and equation of latus rectum through S´ is x = – ae.
(v) SA. SA¢ = CB2. We have SA = CA – CS = a – ae and SA¢ = CS + CA = a + ae.
So SA.SA´ = (a – ae) (a + ae) = a2 (1 – e2) = b2 = CB2.

MATHS 4.43
(vi) SB + SB¢ = 2a. We know co-ordinates of B and B’ are respectively (0, b) and (0, – b).
Co-ordinates of S are (ae. 0)
CO- ORIDINATE GEOMETRY

SB = (ae - 0)2 + (0 - b )2 = a 2 e 2 + b 2 = a 2 = a.

2 2
SB¢ = (ae - 0) +S(B0¢ += b )(ae= - 0a)22 =
10.1.1.1.1.1.1.1.1
2
+ a(0 + b ) = a 2 = a

10.1.1.1.1.1.1.1.2 \ SB + SB¢ = a + a = 2a,

Observations :
x 2 y2 x 2 y2
If b = a then + = 1 becomes + =1
a 2 b2 a2 a2
Or, x2 + y2 = a2 a circle. So a circle is a special formation of an ellipse.
An important Property :
The sum of focal distance of any point on an ellipse is equal to the major axis.
DIFFERENT FORMS OF THE EQUATION OF ELLIPSES :
A. Focus lying on y-axis
Taking focus S on the y-axis and directrix L parallel to x-axis we can get a different form of the
equation of ellipse (proceeding similarly as standard equation) where x and y are interchanged. The
equation is as follows :
y2 x 2
+
a 2 b2
( )
= 1, a 2 > b 2 . Where the centre is at the origin, major axis along the y-axis and minor axis

along the x-axis. The equations, of directrices would be y = a ± e.


An important Note :
x 2 y2
For the equation of the form + = 1. If A2 > B2, then major axis is along the x-axis, if again
A 2 B2
A2 < B2, then major axis is along the y-axis.
Alternative way :

Y
Z M L
A
S P

y
C B
X
x

4.44 MATHS
It can be derived independently. Focus S is the point (0, ae).
Since SP2 = e2 PM2 we get,
2
x2 y2
(x - 0)2 + (y - ae )2 = e 2 éê a - y ùú ; or, + = 1 (on reduction)
ëe û (
a 2 1 - e2 )a2

x 2 y2
or + = 1 [where b2 = a2 (1 – e2)]
b2 a 2
Observation :
Here the foci are (0, ± ae ) and equations of directrices are y = ± a / e.
B. Centre is at (h, k)
Let the centre of the ellipse C be at the point (h, k). The major axis of length 2a parallel to the axis
and the minor axis of length 2b parallel to the y-axis.
x 2 y2
Now with CX¢ and CY¢ as as axes of co-ordinates, the equation to the ellipse is + = 1.
a 2 b2
Now the point C being (h, k), the co-ordinates of O referred to CX¢ and CY¢ as axes are (– h, –k).
So to get the equation of ellipse with reference to OX and OY as axes of co-ordinates. We are to

replace x by x – h, and y by y – k so the resulting equation is


(x - h )2 + (y - k )2 ( )
= 1, a 2 > b 2 .
a2 b2
Y

C

k y
h
O X
x N

Corr. For major axis parallel to y-axis and centre at (h, k), the equation of an ellipse is

(x - h )2 + (y - k )2 =1
b2 a2
Summary :
x 2 y2
(A) For the equation of an ellipse + = 1 (a > b ). (refer the first figure)
a 2 b2
1. CA = a, CB = b, CS = ae, CZ = a/e
2. Co-ordinates of centre (C) ; (0, 0)
3. Co-ordinates of vertices A, A¢ : (± a, 0)
4. Co-ordinates of B, B¢ : (0, ± b)
MATHS 4.45
5. Co-ordinates of Z, Z¢ : (± a/e, 0)
6. Co-ordinates of foci (S, S¢) : (± ae, 0)
CO- ORIDINATE GEOMETRY

7. Length of major axis = 2a


8. Length of minor axis = 2b

9. Length of latus rectum = 2b 2 / a


1 2
10. e = a - b2
a
11. equation of major axis : y = 0
12. equation of minor axis : x = 0
13. equation of directrices : x = ± a / e
14. equation of latus recta : x = ± ae
15. distance (SS¢): 2ae.

y2 x 2
B. For the equation of an ellipse + = 1, (a > b)
a 2 b2
1. co-ordinates of centre : (0, 0)
2. co-ordinates of vertices : (0, ± a)
3. co-ordinates of foci : (0, ± ae)
4. length of major axis : 2a
5. length minor axis : 2b
6. length latus rectum : 2b2/a
7. ecentricity is given by b2 = a2 (1 – e2)
8. equation of major axis : x = 0
9. equation of minor axis : y = 0
10. equation of directrices ; y = ± a / e
11. equation of latus recta : y = ± ae
12. distance of foci = 2ae.
SOLVED EXAMPLES :
1. Find :
(i) co-ordinates of vertices and foci ;
(ii) length of major axis, minor axis and latus rectum ;
(iii) equations of directrices, latera recta and
(iv) distance of foci of the ellipse
x 2 y2 x 2 y2
(a) + = 1. (b) + = 1.
16 9 9 16
5. Comparing the given equation with the standard equations of an ellipse x2/a2 + y2/b2 = 1, we get
a = 4, b = 3. Here a > b, so major axis is along the x-axis.

4.46 MATHS
1 2 1 1
(a) e = a - b2 = 16 - 9 = 7 (e is not negative as 0 < e < 1) centre (0, 0)
a 4 4
(i) Co-ordinates : vertices (± a, 0) = (± 4, 0) ;

é 7 ù
foci (± 4,0) = ê± 4, , 0ú = ( ± 7, 0)
êë 4 úû
(ii) length : major axis = 2a = 2.4 = 8 : minor axis = 2b = 2.3 = 6
latus rectum = 2b2 / a = 2.9/4 = 9/2 = 4.5 unit
4 16
(iii) equations : directrices x = ± a/e or x = ± or, x = ±
7 /4 7

7
latera recta x = ± ae or x = ± 4. or x = ± 7
4

7
(iv) distance of foci = 2ac = 2.4 =2 7 .
4
(b) In this case the major axis is along the y-axis, minor axis along the x-axis. So the vertices and the
foci will be on y-axis and latera recta and directrices are parallel to x-axis, centre is at
origin (0, 0).
1 2 1 1
Here a2 = 16, b2 = 9 i.e., a = 4, b = 3, e = b - a2 = 16 - 9 = 7
b 4 4
(iii)
(i) co-ordinates of vertices : (0, ± a) = (0, ± 4) ;

(
foci (0, ± ae) = 0, ± 4. 7 / 4 )
(ii) length : major axis = 2a = 2 ´ 4 = 8,
(iv)
minor axis = 2b = 2 ´ 3 = 6.
9
Latus rectum = 2b2/a = 2.9/4 = = 4.5 unit
2

7
(i) equations : directrices y = ± a/e or, y = ± 4.
4

7
(ii) distance of foci = = 2ae = 2.4. =2 7
4
2. Find equation of the ellipse whose foci are (3, 0) and ( – 3, 0) and the eccentricity is 2/3
x 2 y2
Let the euqation of the ellipse be + =1;
a 2 b2
Co-ordinates of foci are ( ± ae, 0).
2 9 81
Now ae = 3 or, a. = 3 or, a = i.e., a 2 =
3 2 4
81 æ 4 ö 81 5 45
(
Again, b 2 = a 2 1 - e 2 = ) ç1 - ÷ = . =
4 è 9ø 4 9 4
,

MATHS 4.47
x 2 y2
\ reqd. equation is + = 1 or, 20x2 + 36y2 = 405.
81 45
CO- ORIDINATE GEOMETRY

4 4

2
3. Find the equation of the ellipse passing through the point (– 3, 1), having eccentricity .
3

x 2 y2
Let the equation of the ellipse + = 1. Since it passing through the point (–3, 1) we get
a 2 b2
9 1
+ =1 …. (i)
a 2 b2
3
æ 2ö a
( )
Again b 2 = a 2 1 - e 2 = e 2 ç1 - ÷ = ….(ii)
è 3ø 3
Solving (i), (ii) a2 = 12, b2 = 4.
x 2 y2
\ required equation is + = 1 or x2 + 3y2 = 12.
12 4
4. Find the equation of the ellipse whose length of latus rectum is 8 units and that of the semi-major
axis is 9 units.
x 2 y2
Let the equation to the ellipse be + = 1.
a 2 b2

2b 2 2b 2
Here a = 9 and length of latus rectum = = 8 or, b2 = 36
a 9

x 2 y2
\ required equation is + =1
81 36

x 2 y2
5. The eccentricity of the ellipse + = 1 is same to that of the ellipse
169 25

x 2 y2
+ = 1. Find the ratio of a to b. [ICWA (F) Dec. 2000]
a 2 b2
From 1st ellipse, ellipse, 25 = 169 (1 – e2) where e is eccentricity.
25 25 144 12
\ 1– e2 = or e 2 = 1 - = or e =
169 169 169 13

æ 144 ö 25
From 2nd ellipse, b 2 = a 2 ç1 - ÷ or b 2 = a 2 .
è 169 ø 169

a 2 169 a 13
Or, = or, = .
b2 25 b 5
6. Find the equation of an ellipse for which the principal axis are along the axes of the co-ordinates,
3
the foci are (0, ± 3) and eccentricity is .
4
Here the foci of the ellipse lie on y-axis, its centre is at origin. So the major axis is along the y-axis.

4.48 MATHS
x 2 y2
Let the equation of the ellipse is + = 1, a > b.
b2 a 2
3
Now e = ; co-ordinates of foci (0, ± ae) = (0, ± 3)
4
3
\ ae = 3 or a = 3 or a = 4,
4

æ 9 ö 7
Also b2 = a2 (1 – e2) or b 2 = 16 ç1 - ÷ = 16. = 7
è 16 ø 16

x 2 y2
\ the reqd. equation is + = 1.
7 16
7. Find the equation of the ellipse referred to its axes as axes of co-ordinates, whose vertices are
16
(± 4, 0) and the directrices are x = ± .
7

x 2 y2
Let the equation be + = 1. Co-ordinates of vertices ( ± 4, 0) i.e., a = 4.
a 2 b2
16
Again eqn. of directrices are x = ±
7

a 16 4 16 7
i.e, = or, = , or e = .
e 7 e 7 4

æ 7 ö 9
We know b2 + a2 (1 – e2) or b 2 = 16 ç1 - ÷ = 16. = 9
è 16 ø 16

x 2 y2
\ the reqd. equation is + =1
16 9
8. For the ellipse 9x2 + 16y2 – 54x + 64y + 1 = 0.
Find the :– (i) co-ordinates of centre, vertices and foci
(ii) eccentricity (iii) latus rectum (iv) equation of axes and directrices
(v) lengths of major and minor axes (vi) distance between foci.
Arranging the given equation, we get
9(x2 – 6x + 9) + 16(y2 + 4y + 4) = 144

or, 9(x – 3)2 + 16(y + 2)2 = 144 or


(x - 3)2 + (y + 2)2 =1
16 9

X2 Y2
or, + = 1, where X = x – 3, Y = y + 2, a2 = 16, b2 = 9 (here a2 > b2).
16 9
The eccentricity is given by

7 7
b2 = a2 (1 –e2) or, 9 = 16 (1 – e2) or, e 2 = or, e =
16 4
(i) co-ordinates of centre aer X = 0, Y = 0
i.e. x – 3 = 0 or, x = 3 and y + 2 = 0 or, y = – 2.
MATHS 4.49
So centre is at the point (3, – 2)
For vertices, co-ords. are ( ± a, 0) i.e. ( ± 4, 0)
CO- ORIDINATE GEOMETRY

Co-ordinates of foci are (± ae, 0) i.e. X = ± ae, Y = 0

7
or, x – 3 = ± 4
4

or, x = 3 ± 7 and y + 2 = 0 i.e. y = – 2

(
\ reqd. co-ords of foci are 3 ± 7 ,-2 )
7
(ii) e= (shown before)
4

2b 2 9 9
(iii) latus rectum = = 2 = = 4.5 units
a 4 2
(iv) Equation of major axis Y = 0 i.e. y + 2 = 0
Equ. of minor axis is X = 0 i.e. x – 3 = 0

7
Equs. of directrices are X = ± a / e or x – 3 = ±
4
16
or x = 3 ±
7
(v) Length of major axis = 2a = 2.4 = 8
Length of minor axis = 2b = 2.3 = 6
Note. Length of semi-major axis = 4, and that of semi-minor axis = 3.

7
(vi) Distance between foci = 2ae = 2.4 =2 7
4
9. Find the equation of the ellipse referred to its axes as the axes of x and y respectively, which passes
through the point (– 3, 1) and (2, – 2). Find also its eccentricity and latus rectum.
x 2 y2
Let + = 1 be the equation of the ellipse.
a 2 b2
9 1
Since it passes through (– 3, 1), (2, – 2) we get 2
+ 2 =1 …. (i)
a b
4 4
2
+ 2 =1 … (ii)
a b
32 32
Solving (i), (ii) we get a2 = , b 2 = , (a2 > b2)
3 5

3x 2 5y 2
The eqn. of the ellipse is + = 1 or, 3x2 + 5y2 = 32
32 32
32 32 2
10.1.1.1.1.1.1.1.1 Again b2 = a2 (1 – e2) or, = (1 - e2 ) or, e 2 = or, e = 2 / 5
5 3 5

4.50 MATHS
b2 32 3 8
Latus rectum = 2 =2 = 6 = 3.918.
a 5 32 5
1
10. Find the equation of the ellipse whose focus is (– 1, 1) eccentricity is and the directrices is
2
x – y + 3 = 0. [ICWA (F) June ’98]
Let P (x1, y1) be a point on the ellipse.

The distance of P(x1, y1) from focus S(–1, 1) is (x 1 + 1)2 + (y1 + 1)2 and the perpendicular distance

x 1 - y1 + 3 1
from P on directrices x –y + 3 = 0 (as distance = PM) is given by ;e= .
2 2
We have, SP = e.PM
2 2 1 x1 - y1 + 3
or, (x1 + 1) + (y1 - 1) =
2 2
1
or, (x 1 + 1)2 + (y1 - 1)2 = (x 1 - y1 + 3)2
4. 2
or, 8x12 + 16x1 + 8 + 8y12 – 16y1 + 8 = x12 + y12 + 9 – 2x1y1 – 6y1 + x1
or, 7x12 + 7y12 + 2x1y1 + 10x1 – 10y1 + 7 = 0
so locus of P is (x1, y1) is 7x2 + 7y2 + 2xy + 10x – 10y + 7 = 0.
11. Show that the point (5, 2) lies outside the conic 2x2 + 7y2 = 14
x 2 y2
The conic is + = 1.
7 2
Now putting (5, 2) in the L.H.S. of the conic we get
25 4 4 4
+ = 3 + 2 = 5 >1
7 2 7 7
Hence the given point lies outside the given conic.
12. The ellipse px2 + 4y2 = 1 passes through the points (± 1, 0), Find the value of p and hence find the
lengths of its major and minor axes. [ICWA (f) June 2005]
Since the ellipse px + 4y = 1 passes through the point ( ± 1, 0), so we have p. (± 1)2 + 4 ´ 0 = 1
2 2

x2 y2 1
or, p = 1. So eqn. is + = 1, a 2 = 1, b 2 =
1 1 4
4
1
Length of major axis = 2a = 2 ; that of minor axis = 2b = 2. =1.
2

13. The major and minor axes of an ellipse are the x and y axes respectively. Its eccenticity is 1
2
and the length of the latus rectum is 3 units. Find the equation of the ellipse. [ICWA (F) June 2006]

1 2 1 1 2 1 1 b2 b2 1
e= a - b 2 or, = (
a - b 2 or, = 2 a 2 - b 2 = 1 - 2 or , 2 = )
a 2 a 2 a a a 2

2b 2
or, 2b2 = a2 …… (i) Again length of latus vectum = = 3 or, 2b2 = 3a ….. (ii)
a
MATHS 4.51
From (i), (ii), a2 = 3a or, a(a – 3) = 0, or, a = 3, 0 (inadmissible)

From 2b2 = 9 or, b2 = 9


2
x 2 y2 x 2 2y 2
\ reqd. equ. is + = 1 or, + = 1 or, x2 + 2y2 = 9.
9 9 9 9
2

SELF EXAMINATION QUESTIONS :


1. For each of the following ellipses, find the
(i) lengths of major and minor axes
(ii) the length of latus rectum
(iii) the co-ordinates of centre, vertices, foci
(iv) equations of directrices and later recta.

x 2 y2 x 2 y2
(a) + =1 (b) + =1
25 16 16 25
(c) x2 + 2y2 + 2x – 8y + 2 = 0
25
[Ans. (a) : (i) 10, 8 (ii) 12.5 units (iii) (0, 0), ( ± 5, 0), ( ± 3,0) (iv) x ± , x = ± 3.
3
35
(b) (i) 10, 8 (ii) 12.5 (iii) (0, 0), (0, ± 5) (0, ± 3) (iv) y = ± , y=±3
3

(c) (i) 2 7 , 14 (ii) ( )(


7 , (iii) (– 1, 2), 1 ± 7 , 2 , - 1 ± 7 / 2 , 2 )
(iv) x = -1 ± 114 , - 1 ± 7 / 2 . ]

1
2. Find the co-ordinates of foci and the eccentricity of the ellipse : 4x2 + 5y2 = 20. [Ans. (± 1, 0) ; ]
5
3. For the ellipse x2/3 + y2/2 = 1. Find the equation of the directrices. [Ans. x = + 3]

4. Find the distance between foci of the ellipse 3x2 + 4y2 = 24 [Ans. 2 2 ]
2 2
5. (i) For the ellipse x /36 + y /20 = 1, find the eccentricity and length of latus rectum.
[Ans. 2/3, (± 4, 0), x = ± 9]
(ii) For the ellipse x2/25 + y2/16 = 1, find the co-ordinates of foci, eccentricity and length of latus
2
rectum. [Ans. (± 3,0 ), , 6.4 units.]
3
6. Find the equation of the ellipse, referred to its axes as axes of co-ordinates.
(i) which passes through the points (2, 2) and (3, 1)

5
( )
(ii) whose foci are the points ± 5 , 0 and the eccentricity is
3
.

2
(iii) whose latus rectum is 5 and eccentricity is
3
æ 1 ö 5
(iv) whose vertices are ç ± , 0 ÷ and the directrices are x = ± .
è 4 ø 12
[Ans. (i) 3x2 + 5y2 = 32 (ii) x3/9 + y2/4 = 1
4.52 MATHS
(iii) 20x2 + 36y2 = 405 (iv) 16x2 + 25y2 = 1]
7. Find such an equation of ellipse whose
1
(i) eccentricity is and latus rectum is 8. [Ans. 8x2 + 9y2 = 162]
3
(ii) major axis is 12 and minor axis is 8. [Ans. 9x2 + 25y2 = 1]
(iii) major axis is 2/3 and eccentricity is 4/5 [Ans. 9x2 + 2y2 = 1]
1
(iv) foci are (4, 0) and (– 4, 0) and eccentricity is [Ans. x2/64 + y2/48 = 1]
2
(v) foci are ( ± c, 0) and major axis is 2. [Ans. x2 + 2y2 = 2]
(vi) foci are (± c, 0) and major axis is 2a. [Ans. x2/a2 + y2/a2 – c2 = 1]

5
(vii) foci are (0, ± 5) and eccentricity . [Ans. 9x2 + 4y2 = 36]
3

(viii) ( )
foci are 0, ± 5 and eqn. of directrices are y = ± 9 / 5 . [Ans. 9x2 + 4y2 = 36]

(xi) vertices are ( ± 5, 0) and one focus is (4, 0) [Ans. x2/25 + y2/9 = 1]
(x) vertices are (± 4, 0) and minor axis is 4. [Ans. x2/16 + y2/4 = 1]
8. Find the equation of an ellipse whose principal axes are along the co-ordinates axes, whose
eccentricity is ½ and distance between foci is 4. Show also that the curve passes through (2, 3)
[Ans. x2/16 + y2/12 = 1]
9. Find the equation of the ellipse whose

(i) focus is (6, 7), eccentricity is 1 / 3 and directrix is x + y +2 = 0.


1
(ii) focus is (2, – 3), eccentricity is and directrix is x – 2y – 15 = 0.
2

(iii) focus is (– 1, 1) eccentricity is 1 / 2 and directrix is x – y + 4 = 0.


[Ans. (i) 5x2 + 5y2 –76x – 88y – 2xy + 506 = 0.
(ii) 19x2 + 16y2 + 4xy – 50x + 60y + 35 = 0
(iii) 3x2 + 3y2 + 2xy – 8 = 0.]
10. The latus rectum of an ellipse is 4 inches and distance of vertex from the nearer focus is 1.5
inches. Find the eccentricity of the ellipse. [Ans. 1/3]
2 2
11. Show that point (5, 3) lies outside the ellipse 2x + 7x = 14]
x 2 y2 1
12. The length of the latus rectum of the ellipse 2
+ 2 = 1 is times the length of its major axis.
a b 3

2
Find its eccentricity. [ICWA (F) June 2000] [Ans. ]
3

2b 2 1
[Hints : = .2a & etc.]
a2 3

MATHS 4.53
OBJECTIVE QUESTIONS :
2
+y
2
1. For the ellipse x = 1 ; find the major and minor axes. [Ans. 8 ; 6]
CO- ORIDINATE GEOMETRY

16 9
9
2. For the above ellipse find the length of latus rectum. [Ans. units]
2
3. Find the eccentricity of the ellipse whose minor axis is half of its major axis. [ICWA (F) Dec. 2006]

3
[Ans. ]
2
1
4. Find the eccentricity of the ellipse 8x2 + 9y2 = 288 [ICWA (F) Dec. 2005] [Ans. ]
3

x 2 y2 2
5. Find the eccentricity of the ellipse + =1 [ICWA (F) Dec. 2004] [Ans. ]
36 20 3
2 y2
6. Find the coordinates of foci of the ellipse x + =1 [Ans. (±1, 0 ) ]
5 4

7. For the above ellipse tid major and minor axis. [Ans. 2 5, 4 ]

2 y2
8. Find the ellipse x + = 1. Find the distance between the foci. [Ans. 8 units]
36 20
4.6 HYPERBOLA :
Definition :
A hyperbola is the locus of a point which moves in a plane such that its distance from a fixed point
point (called focus) lying in that plane bears a constant ratio e (e > 1) to its perpendicular distance
from a fixed straight line (known as directrix).

K´ Y K

L(ae,0)
M´ M P

A´ Z´ Z A (a,0) X
S´(–ae,0) (–a,0) N S(ae,0)


x+ ae = 0

x– ae = 0

Fixed point is focus, straight line is directirx and constant ratio e ( > 1) is known as eccentricity.
Let the focus of hyperboal be S, directrix ZK and eccentricity e ( >1). From S, SZ is drawn
perpendicullar to ZK. SZ is divided in the ratio of e : 1 internally at A and externally at A´.
4.54 MATHS
SA SA¢
Now = = e. A and A´ are two points on the hyperbola. Let C be the midpoint of AA´ ; draw
AZ A¢Z
CY perpendicular to AA´ at C. Let us take C as origin (0, 0), CX as x-axis. CY as Y axis.
x 2 y2
Standard form of equation is
a 2
b
( )
- 2 = 1, where b 2 = a 2 e 2 - 1 . The detail is not shown at present.

Few Definitions :
Vertex. It is the intersecting point of the hyperbola with the straight line passing through focus and
perpendicular o the directrix.
The hyperbola intersects the x-axis at A (a, 0) and A´ (– a, 0) points. These two points A, A´ are the
vertices (See Fig.)
Centre. The midpoint of the line joining the vertices of a hyperbola is known as centre. The midpoint
of AA¢ i.e., C is the centre.
Transverse axis. The line joining the vertices of the hyperbola i.e., AA¢ is the transverse axis. Its
length is 2a.
Conjugate axis. The points B and B¢ on the y-axis are taken such that CA = CB¢ = b. Now BB´ is the
conjugate axis. Its length is 2b.
Latus Rectum. The chord passing through focus and parallel to the directrix is known as latus rectum.
There are two latus rectums.
Y-axis does not intersect the hyperbola in any point. We do not find any real values of y or putting
x = 0 in the equation. B and B´ are not points on hyperbola.
LATUS RECTUM :
Definition :
The chord passing through one of the two foci of the hyperbola and parallel to directrix (or
perpendicular to transverse axis x) is known as latus rectum.
In double ordinate passing through focus is also called latus rectum.
In Fig. LSL¢ is the latus rectum. LS = 1, so the coordinates of L are (ae, 1) lies on the curve, we get

a 2 e 2 12
- 2 = 1 (putting x = ae, y = b in the standard equation)
a2 b

12 b2 æ b2 ö
or
b2
= e 2
- 1 =
a 2 çè
( )
ç as b 2 = a 2 e 2 - 1 or e 2 - 1 = ÷
a 2 ÷ø

b4 b2
or 12 = or 1 = ±
a2 a
æ b2 ö æ b2 ö
\ The co-ordinates of L and L¢ are ç ae, ÷ and ç ae - ÷
è a ø è a ø

b2
Length of latus rectum = LSL¢ = 2LS = 2
a

MATHS 4.55
2b 2
Note : Length of every latus rectum =
a
CO- ORIDINATE GEOMETRY

To find the equation of a hyperbola if


focus (a, b), directrix (ax + by + c = 0) and eccentricity e (> 1) are given.
Let P (x, y) be a point on hyperbola. Now SP = e.PM
or SP2 = e2. PM2 (S is focus, PM is directrix)

or (x - a )2 + (y - b )2 = e 2 .
(1x + my + n )2 , which is the reqd. equation.
12 + m 2
Example : Find the equation of a hyperbola whose focus is (3, 1), directrix 2x + y + 1 = 0 and

eccentricity 2.
Let P(x, y) be a point on the curve, S is focus and PM is drawn perpendicular to the diectrix. By
definition,
SP = e.PM or SP2 = e2.PM2

(x - 3)2 + (y - 1)2 = e 2 .æçç 2 x + x + 1 ö÷÷


è 4 +1 ø

or (x - 3)2 + (y - 1)2 = 2.
(2x + y + 1)2 which is the reqd. equation.
2
DIFFERENT FORMS OF THE EQUATION OF HYPERBOLA :
(i) If the centre of hyperbola is at (0, 0) focus lying on y-axis and directrix parallel to x-axis,
y2 x 2
then the equation of the hyperbola will be + = 1. Here transverse axis is y-axis
a 2 b2
and conjugate axis will be x-axis.
(ii) If the centre is at (a, b), transverse axis and conjugate axis are respectively x and y
axes, then the equation will be

(x - a )2 - (y - b)2 = 1.
a2 b2
(iii) Centre is at (a, b ) and transverse axis and conjugate axis are respectively y and x
axis, then the equation will be
2 2
(y - b ) (x - a )
- = 1.
a2 b2
(iv) Taking focus S (ae, 0) as origin, SX as x-axis and perpendicular SL passing S as y-axis
the equation will be

(x + ae )2 -
y2
=1
a2 b2
(v) Taking vertex A (a, 0) as origin, AX as x-axis and the perpendicular at A as y-axis, the
equation will be

4.56 MATHS
(x + a )2 -
y2
=1
a2 b2

æa ö
(vi) Z ç , 0 ÷ as origin, ZX at x-axis and directrix ZM as y-axis the equation will be
èe ø
2
æ aö
çx + ÷
è eø y2
- = 1.
a2 b2
The difference of the focal distance of any point lying on the hyperbola is constant and is equal to the
length of transverse axis.
Let P(x1, y1) be any point on the curve, (from figure given).
æ aö
Now SP = e.PM = e.NZ = e(CN – CZ) = e ç x1 - ÷ = ex1 - a
è eø

æ aö
And S¢P = e.PM¢ = e.NZ¢ = e (CN + CZ) = e ç x 1 + ÷ = ex1 + a
è eø

S´P – SP = (ex1 + a) – (ex1 – a) = a + a = 2a = constant.


Again, S´P – SP = 2a = length of transverse axis = AA¢
CONJUGATE HYPERBOLA :
If the transverse axis and conjugate axis of a hyperbola are respectively the conjugate and transverse
axis of another hyperbola, then one of the two hyperbolas is called conjugatehyperbola of the other.
RECTANGLAR (OR EQUILATERAL) HYPERBOLA :
A hyperbola having its transverse axis and conjugate axis equal to one another, is called rectangular
hyperbola.
x2 y2 x2 y2
In the eqn. 2
- 2
= 1 putting a = b, we get 2
- 2 = 1 or x2 – y2 = a2 which is the required
a b a a
equation.
We know, b2 = a2 (e2 – 1) ;
So in the case a2 = a2 (e2 – 1)

or, e2 – 1 = 1 or e2 = 2 or e = 2.
POSITION OF A POINT IN RELATION TO HYPERBOLA :
Any point P(x1, y1) will be outside, upon or inside the hyperbola

x2 y2 x 1 2 y1 2
- = 1 or, - 2 = 1 , < = > 0.
a 2 b2 a2 b

x2 y2
Example : Find the position of the point (3, – 2) in relation to - = 1.
4 9
2
32 (-2 )
By definition the point (3, – 2) will lie outside, upon or inside hew
the hyperbola if - - 1. is
4 9
negative, zero or positive.
MATHS 4.57
9 4 81 - 16 - 36 81 - 52 29
Now, - -1 = = = > 0 i.e., positive
4 9 36 36 36
CO- ORIDINATE GEOMETRY

\ (3, – 2) will lie inside the hyperbola.

x 2 y2 y2 x 2
A. In the hyperbola - = 1. - =1
a 2 b2 b2 a 2
(i) co-ords. of centre (0, 0) (0, 0)
(ii) transverse axis is on x-axis y-axis
(iii) conjugate axis is on y-axis x-axis
(iv) eqn. of transverse axis y=0 x=0
(v) eqn. of conjugate axis x=0 y=0
(vi) length of trans. Axis 2a 2b
(vii) length of conjugate axis 2b 2a
(viii) co-ords. of vertices (± a, 0) (0, ± b)
(ix) co-ords of focii (± ae, 0) (0, ± be)
a b
(x) equation of of directrices x=± y=±
e e

b2 a2
(xi) ecentricity e2 = 1 + e2 = 1 +
a2 b2

2b 2 2a 2
(xii) length of latus rectum
a b
(xiii) equation of latus rectum x = ± ae y = ± be
(xiv) distance between foci 2ae 2be
(xv) distance between directrices 2 a/e 2 b/e

B. In the parabola :
(x - a )2 - (y - b)2 = 1.
a2 b2
(i) Co-ords. of centre (a, b ).
(ii) trans. Axis and conjugate axis are parabell to x and y axes respectively.
(iii) equation of trans. Axis : y – b = 0.
Equation of conj. Axes : x – a = 0.
(iv) length of trans. axis 2a : length of conj. axis : 2b.
(v) co-ords. of vertices : ( a ± ae, b).
(vi) co-ords. of focii : (a ± ae, b).

4.58 MATHS
2b 2
(vii) length of latus rectum :
a

b2
(viii) eccentricity e is defined by e 2 = 1 + .
a2
a
(ix) eqn. of directrices : x = a ± .
e

x 2 y2 y2 x 2
C. - = 1 and - = 1 are conjugate hyperbolas. If e1 and e2 are the respective
a 2 b2 b2 a 2
eccentricities, then

( )
b2 = a 2 e2 - 1 ; a 2 = b2 e2 - 1 ( )
D. Equation of rectangular Hyperbola : x2 – y2 = a2
(i) trans. and conj. axis are respectively x and y axes
b axis
. (ii) length of trans. axis = length of conj. axis = 2a.

(iii) e = 2
2 2
x y
E. In - = 1, a point P(x1, y1) will lie outside, upon or inside if
a 2 b2
2 2
x1 y
2
- 12 = 1 < = > 0.
a b

F. In
(y - b )2 - (x - a )2 = 1.
a b2
(i) co-ords. of centre : (a, b).
(ii) trans. axis and conj. axis are parallel to y and x axes respectively.
(iii) eqn. of trans. axis : x – a = 0 : eqn. of conj. axis : y – b = 0.
(iv) length of trans. axis = 2a ; length of conj. axis = 2b.

SOLVED EXAMPLES :
1. In the hyperbola 4x2 – 9y2 = 144 ; find (I) co-ordinates of vertices, (ii) eccentricity, (iii) co-
ordinates of foci, (iv) length of latus rectum, (v) equation of latus rectum, (vi) equation of
directrices, (vii) length of axes.

x2 y2
From 4x2 – 9y2 = 14 we get - = 1.
36 16
Comparing this with the standard equation of hyperbola, we find a2 = 36 ; b2 = 16 ; or a = 6, b = 4.
(i) co-ordinates of vertices : (± a, 0) or (± b, 0).

b2 16 52
(ii) eccentricity : e 2 = 1 + =1+ =
a2 36 36

52 2 13 13
or, e = = = .
36 6 3
MATHS 4.59
4.60
CO- ORIDINATE GEOMETRY
(v)

MATHS
2b 2 9
Length of latus rectum : = 2. = 09.
n 2

b2 9 13
(ii) eccentricity : e = 1 + 2
= 1+ =
a 4 2
4. Find the coordiantes of centre, length of latus rectum, eccentricity, and the coordinates of foci of the
hyperbola 3x2 – 4y2 – 12x – 8y – 4 = 0 [ICWA (F) Dec. 2005]
2 2 2 2
3x – 4y – 12x – 8y – 4 = 0 or, 3(x – 4x + 4) – 4 (y + 2y + 1) = 12
or, 3(x – 2)2 – 4 (y + 1)2 = 12 or, 3(x – 2)2 – 4(y – (– 1)2 = 12

or,
(x - 2 )2 - (y - (- 1))2 = 1. Comparing with
(x - a )2 - (y - b)2 = 1 we get
4 3 a2 b2

a = 2, b = -1, a = 2, b = 3.

Centre : (a, b ) i.e. (2,-1)

b2 3
length of latus rectum : 2 i.e. 2 ´ = 3
a 2

b2 3 7 7
eccentricity : e 2 = 1 + 2
=1+ = ; e =
a 4 4 2

æ 7 ö÷
foci : (a, ± ae) = ç 2 ± 2.
ç 2 ÷ø
(
= 2+ 7 )
è

5. Find the equation of the hyperbola whose focus is at (1, 2) directrix 2x + y = 1 and eccentricity 3.
[ICWA (F) Dec. 2006]
Let P(x, y) be a point on hyperbola. Draw PM perpendicular on the directrix. S is focus, so SP = e.PM
or, SP2 = e2 PM2
2 2
2 2æ 2x + y - 1 ö
or, (x – 1) + (y – 2) = e . çç 2
÷÷ or, (x – 1)2 + (y – 2)2 = 3 ´
(2x + y - 1)
è 4 +1 ø 5
or, 5(x2 – 2x + 1 + y2 – 4y + 4) = 3 (4x2 + y2 + 1 + 4xy – 4x – 2y)
or, 7x2 – 2y2 + 12xy – 2x + 14y – 22 = 0 (on reduction).
6. Find the intersecting points of the hyperbola x2 – 2y2 = 2 with the straight line x + 2y – 4 = 0 and
find also the length of the chard thus formed.
x2 – 2y2 = 2 …… (i), x + 2y – 4 = 0 …… (ii) solving we get x = 2, y = 1 and x = – 10, y = 7 (to show
the solution is detail)
\ reqd. intersecting points are (2, 1) and (– 10, 7)

Length of chord = (- 10 - 2)2 + (7 - 1)2 = 144 + 36 = 180 = 6 5 units.

7. Taking the axes of a hyperbola as co-ordinate axes, find the equation of a hyperbola in each of the
following cases :-
(i) When length of transverse and conjugate axes are respectively 8 and 10.
MATHS 4.61
æ 2ö
(ii) When the hyperbola passes through the points (5, 0) and ç - 7, ÷.
è 5ø
CO- ORIDINATE GEOMETRY

(iii) For eccentricity 3 and distance between foci 12.


(iv) For distance between foci 13 and conjugate axis 5.

5
(v) For eccentricity and thus rectum 3.
8
(i) Length of transverse axis = 2a = 8 or, a = 4 ;
length of conjugate axis = 2b = 10 or, b = 5.
x 2 y2 x 2 y2
\ required equation is 2
- 2 = 1 or, - = 1.
4 5 16 25

x 2 y2
(ii) Let the required equation be - = 1. Since it passes through (5, 0),
a 2 b2
25
so - 0 = 1 or, a 2 = 25.
a2
æ 2ö
Again the hyperbola passes through ç - 7, ÷ ,
è 5 ø

49 ( 2 / 5) 2 49 4
So 2
- 2
= 1 or - =1
a b 25 25b 2
1
or, b 2 =
6

y2
2
x x2
\ required equation is - 1 = 1 or, - 6 y 2 = 1.
25 6 25

(iii) e = 3 and distance of foci = 2ea = 12 or ae = 6


6 6 36
or, a = = or, a 2 = = 12.
e 3 3

b2 b2 b2
Again e 2 = 1 + 2
or, 3 = 1 + or = 2 or, b2 = 24.
a 12 12

x 2 y2
\ equation is - = 1.
12 24
5
(iv) Here conjugate axis = 2b = 5 or, b = ;
2
13
distance of foci = 2ae = 13 or, e = .
2a
2
æ5ö
b 2
149 ç ÷ 149 25
2
Again, e2 = 1 + 2 or, 2 = 1 + è 2ø or, 2
=1+ 2
a 4a a 4a 4a

4.62 MATHS
1
or, 2
(169 - 25)= 1 or, 1442 = 1 or, 4a2 = 144
4a 4a
or a2 = 36 or, a = 6
x2 y2 x 2 4y 2
\ required equation is - 2 = 1 or, - = 1.
36 (5 ) 36 25
2

5 2 5 2b 2
(v) e = , e = , again latus rectum = = 3
3 3 a

or,
(
2a 2 e 2 - 1) ( ) æ5 ö
= 3 [as b2 = a 2 e 2 - 1 ] or, 2a ç - 1÷ = 3
a è3 ø

2 3 3 27
or, 2a. = 3 or, a = or, a 2 = .
3 4 16

2b 2 a 3 3 9
Form = 3 , we get b 2 = . 3 = . 3=
a 2 2 .4 8

16 x 2 8 y 2
\ required equation is - = 1.
17 9
8. Find the position of the points (i) (1, 1), (ii) (3, – 1) in relation with the hyperbola
x2 y2
- = 1.
4 9
x2 y2 1 1
(i) The point (1, 1) will lie outside, upon or inside the hyperbola - = 1 if - = -1 is
4 9 4 9
negative, zero or positive.
1 1 9 - 4 - 36 -31
Now, - =1= = = a negative number.
4 9 36 36
\ (1, 1) lies outside the hyperbola.
2
32 (- 1) 9 1 81 - 4 - 36 41
(ii) Similarly, - -1 = - -1 = = = a positive number.
4 9 4 9 36 36
So, (3, – 1) lies inside the hyperbola.

SELF EXAMINATION QUESTIONS :


1. Find (i) co-ordinate of vertices, (ii) eccentricity,
(iii) co-ordinates of foci,
(iv) length of latus rectum. (v) equation of latus rectum,
(vi) equation of directrices,
(vii) length of axes, for each of the following hyperbola :-
(a) 4x2 – 9y2 = 36. (b) 3x2 – 4y2 = 48.

13
[Ans. (a) (i) (± 3, 0 ) (ii)
3
( )
(iii) ± 13 , 0 ]

MATHS 4.63
(iv) 3 (v) x = ± 13 (vi) 13x = ± 9 (vii) 6, 4

7
( )
CO- ORIDINATE GEOMETRY

(b) (i) (± 4, 0 ) (ii) (iii) ± 2 7 , 0 (iv) 6 (v) x = ±2 7


2

(v) 7 x = ± 8 (vii) 8, 4 3. ]
2. (a) For the hyperbola 16x2 – 9y2 + 32x + 36y – 164 = 0 find :
(i) co-ordinates of centre, (ii) latus rectum,
(iii) eccentricity,
32 5
(iv) co-ordinates of foci. [Ans. (i) (–1, 2) (ii) (iii) (iv) (- 1 ± 5, 2 )]
3 3
(b) The equation 3x2 – 4y2 – 12x – 8y – 4 = 0 represents hyperbola. Find its centre foci and lengths of

the axes. [Ans. (2, – 1) ; 2 ± 2 7 ,-1 ; 4, 2 3 ]

3. (a) For the hyperbola


(x - 2)2 -
y2
= 1 ; find
16 9
(i) co-ordinates of centre. (ii) latus rectum,
(iii) eccentricity,
9 5
(iv) co-ordinates of foci. [Ans. (i) (2, 0) (ii) (iii) (iv) (2 ± 5, 0 ) ]
2 4
(a) Find the coordinates of the centre, foci and the equations of the directrices of the hyperbola,

(x + 1)2 - (y + 2)2 = 1. [ICWA (F) Dec. 2003] [Ans. 1, – 2 ; –1, ± 5, - 2 ; x = -1 ±


16
]
16 9 5
4. If for a hyperbola the transverse axis i.e. equal to the latus rectum, then find the eccentricity of that

hyperbola. [Ans. 2]
5. Find the equation of the hyperbola, referred to its axes as axes of co-ordinates and whole :
(i) transverse axis and conjugate axis are respectively 4 and 6. [Ans. 9x2 – 4y2 = 36]
6
(ii) eccentricity is and transverse axis is 10. [Ans. 11x2 – 25y2 = 275]
5

5
(iii) eccentricity is and distance between foci is 5. [Ans. x2 – 4y2 = 1]
2

3
(iv) eccentricity is and length of semi-latus rectum is. [Ans. x2 – 2y2 = 16]
2
(v) transverse axis is 4 and length of semi latus rectum is 8. [Ans. 4x2 – y2 = 16]
(vi) length of transverse axis is 8 and pass through (– 5, 2) [Ans. 4x2 – 9y2 = 64]
(vii) transverse axis is 6 and vertex is the midpoint of centre and focus.
[Ans. 3x2 – y2 = 27]
(viii) focii are ( + 5, 0) and eccentricity is 5. [Ans. 24x2 – y2 = 24]
5
(ix) eccentricity is and co-ordinate of one focus is (5, 0). [Ans. 9x2 – 10y2 = 90]
4
4.64 MATHS
(x) co-ordinates of vertices and foci are respectively (± 4, 0) and (± 6, 0)
[Ans. 5x2 – 4y2 = 80]
(xi) foci are (0, ± 3) and transverse axis is 5. [Ans. 100y2 + 44x2 = 275]

æ 5 ö 9 x2 y2
(xii) focii are ç ± , 0 ÷ and latus rectum is . [Ans. - =1]
è 2 ø 4 4 9/ 4

9 x 2 y2
(xiii) latus rectum and one focus at (5, 0). [Ans. - =1]
2 16 9
8. Find the equation to a hyperbola whose axis are referred to co-ordinates axes and which :
(i) passes through (2, 3) and (4, 7). [Ans. 10x2 – 3y2 = 13]
(ii) passes through (– 3, 2) and transverse axis is 8. [Ans. 4x2 – 5y2 = 16]

(iii) passes through (2, – 1) eccentricity is 3 [Ans. 2x2 – y2 = 7]


9. Find the equation to a hyperbola whose :

(i) focus is at (4, 0) and nearer directrix is 2x + y – 1 = 0 and eccentricity is 2


2 2
[Ans. 3x – 3y + 8xy + 22x + 6y – 48 = 0]
OBJECTIVE QUESTIONS :

1. Find the eccentricity of the hyperbola 9x2 – 16y2 = 144. [Ans. 5 ]


4
1. For the hyperbola x2 – 4y2 = 16 ; find (i) coordinates of focii (ii) length of latus rectum
(iii) coordinates of vertices (iv) equation of directrices.
8
( )
[Ans. (i) ± 2 5 , 0 (ii) 4 units (iii) (± 4, 0) (iv) x = ±
5
]

3. Find the eccentricity of the hyperbola x2 – 2y2 = 2 [Ans. 3 ]


2
2. Find the equation of the rectangular hyperbola whose length of latus rectum is 6 units.
[Ans. x2 – y2 = 9]
x 2 y2
5. Find latus rectum of - =1 [Ans. 9 units]
16 9 2
5. The equations of conic is x2 – y2 = 25. Find the eccentricity and length of latus rectum.

[Ans. 2 ; 10 units]
2 2
7. Find the focii and eccentricity of the hyperbola 9x – 16y = 36.

(
[Ans. ± 5 , 0 ; 5 ]
2 4
)
x 2 y2
8. Determine the eccentricity of the hyperbola - =1 [ICWA (F) June 2007]
25 16

41
[Ans. ]
5

MATHS 4.65
9. Find the coordinates of the centre, foci and the equations of directrices, latus rectum, eccentricity of

(x + 1)2 - (y + 2)2
CO- ORIDINATE GEOMETRY

the hyperbola = 1. [ICWA (F) Dec. 2003]


16 9

[Ans. (–1, 2) ; (4, – 6) ; ± 16 , 9 units ; 5 ]


5 2 4
1. In the hyperbola 25y2 – 16x2 = 400 ; find the equation of latus rectum and length of axes.
10.

[Ans. y = ± 41 ; tran. axis = 8, cong. axis = 10]


BOOKS FOR REFERENCE :
1. Basic Mathematics and Statistice – N. K. NAG.
2. El. Of Co-ordinate Geometry – S.L. loney.
10.1.1.1.1.1.1.1.1 Business Mathematics ISC – N. K. Nag

4.66 MATHS

Vous aimerez peut-être aussi